Blackboard Abdominal EMQs Flashcards

1
Q

A 23 year old woman presents with a 24 hour history of right illiac fossa pain. There is tenderness and guarding in the right illiac fossa. There are no menstrual symptoms. Abdominal and pelvic ultrasound are normal.

What should be the first investigation?

  1. AXR
  2. Laxatives
  3. CT scan
  4. ECG
  5. Oral antibiotics
  6. NG tube
  7. Endoscopy
  8. Ultrasound scan
  9. Laparotomy
  10. Serum amylase
  11. Diagnostic laparoscopy
  12. PR exam
  13. Serum potassium
A
  1. Diagnostic laparoscopy

An abdominal and pelvic CT scan would normally be ordered in situations like this, with possible appendicitis, but it is assumed here that the doctors were thinking of pregnancy as a possibility so a sonogram was done instead, which turns out to be inconclusive. Now, in this situation, you could do an abdominal MRI (especially in early pregnancy) or go ahead with a CT scan anyway, but a diagnostic laparoscopy is the best option here to diagnose and treat at the same time. The main differential here is either obstetric, such as a ruptured ectopic pregnancy, or acute appendicitis. You would have imagined they would have done the usual important tests like FBC and a urinary pregnancy test… but these results are not available, nor are they an option, but a prudent doctor would have ordered them in the diagnostic work up. However, given USS does not show a mass in the fallopian tubes, this may push you away from an ectopic pregnancy… however, USS is operator dependent, this is a female of childbearing age (this age is getting lower and lower in the UK) and the doctor has failed to obtain either serum or urine HCG levels or asked about any missed menstrual periods.

How well did you know this?
1
Not at all
2
3
4
5
Perfectly
2
Q

A 30 year old man presents with severe left loin pain, which is colicky in nature. It radiates to the left groin.

What should be the first investigation?

  1. AXR
  2. Laxatives
  3. CT scan
  4. ECG
  5. Oral antibiotics
  6. NG tube
  7. Endoscopy
  8. Ultrasound scan
  9. Laparotomy
  10. Serum amylase
  11. Diagnostic laparoscopy
  12. PR exam
  13. Serum potassium
A
  1. CT scan

This patient has renal colic which classically presents with severe flank pain radiating to the groin. Infection may complicate renal calculi. Microscopic haematuria is present in up to 90% of cases. Up to 85% of stones are visible on a plain KUB although urate stones are radiolucent. If the stone is radio-opaque, calcification will be seen within the urinary tract. In pregnancy though, a renal USS is first line. The IVP has now been replaced by the CT scan which is the new diagnostic standard. A non-contrast helical (or spiral) CT is preferred due to high sensitivity and specificity and acurately determines presence, site and size of stones. Stones are analysed after they are extracted or when they are expelled to check their composition.

In reality the first test to order is urinalysis – it is quick and simple. You’d also always exclude ectopic pregnancy with a urine pregnancy test if this were a female of childbearing age, look for hypercalcaemia and hyperuricaemia and perform a FBC to look for infection.

How well did you know this?
1
Not at all
2
3
4
5
Perfectly
3
Q

A 45 year old man presents with sudden onset epigastric pain, constant in nature. He has had several previous episodes. He drinks half a bottle of whisky a day.

What should be the first investigation?

  1. AXR
  2. Laxatives
  3. CT scan
  4. ECG
  5. Oral antibiotics
  6. NG tube
  7. Endoscopy
  8. Ultrasound scan
  9. Laparotomy
  10. Serum amylase
  11. Diagnostic laparoscopy
  12. PR exam
  13. Serum potassium
A
  1. Serum amylase

This patient has acute pancreatitis – the cause here being alcohol. He is describing mid-epigastric pain. This pain classically radiates around to the back, which in itself is almost diagnostic. Complicated haemorrhagic pancreatitis may exhibit Cullen’s sign, Grey-Turner’s sign and Fox’s sign. Make sure you know what these are and you are familiar with the causes of acute pancreatitis (GET SMASHED). Those caused by hypocalcaemia may also display Chvostek’s sign and Trousseau’s sign.

Key to diagnosis is serum amylase or lipase levels which are massively elevated. Prognostic criteria are outlined in Ranson’s criteria applied on admission and after 48 hours, or the modified Glasgow score which you can find in your Oxford Handbook. An abdominal CT is however the most sensitive and specific study and findings may include enlargement of the pancreas with irregular contours, necrosis, pseudocysts and peripancreatic fat obliteration.

How well did you know this?
1
Not at all
2
3
4
5
Perfectly
4
Q

A 70 year old woman underwent a right hemicolectomy for caecal cancer 5 years ago. She presents with abdominal distension, colicky pain and profuse vomiting. There is minimal abdominal tenderness.

What should be the first investigation?

  1. AXR
  2. Laxatives
  3. CT scan
  4. ECG
  5. Oral antibiotics
  6. NG tube
  7. Endoscopy
  8. Ultrasound scan
  9. Laparotomy
  10. Serum amylase
  11. Diagnostic laparoscopy
  12. PR exam
  13. Serum potassium
A
  1. NG tube

Bowel obstruction has several causes such as adhesions or cancer in the older patient. In this case the PMH of recent surgery suggests adhesions as a likely aetiology. The profuse vomiting, distended and colicky painful abdomen are indicative. There may also be increased tinkling high pitched bowel sounds. The proximal segment of bowel dilates and distal bowel collapses. Completely obstructed patients generally require surgery. If, on AXR, air is seen to be seeping past the obstruction then the obstruction is partial. As a standard, all patients should be made NBM and given supplemental oxygen, IV fluids and NG decompression (to reduce flow/gastric contents/air towards the obstruction), unless they are rushed off for an emergency laparotomy because, for example, they have complete obstruction and are peritonitic.

How well did you know this?
1
Not at all
2
3
4
5
Perfectly
5
Q

A 46 year old woman presents with a 1 day history of right upper quadrant pain, radiating around the right side of the chest. She says her urine may be darker than usual. Her GP started her on oral antibiotics. Amylase has already been ordered.

What should be the first investigation?

  1. AXR
  2. Laxatives
  3. CT scan
  4. ECG
  5. Oral antibiotics
  6. NG tube
  7. Endoscopy
  8. Ultrasound scan
  9. Laparotomy
  10. Serum amylase
  11. Diagnostic laparoscopy
  12. PR exam
  13. Serum potassium
A
  1. Ultrasound scan

Abdominal ultrasound is ordered when a patient presents with biliary pain and is the single best test for cholelithiasis (though has a low sensitivity for choledocholithiasis). Note that cholelithiasis refers to stones in the gallbladder and choledocholithiasis refers to stones in the bile duct. If stones are found then this would give weight to a diagnosis of acute cholecystitis. There are symptoms here of obstructive jaundice due to gallstone obstruction of bile outflow. Serum amylase would also be ordered in any patient presenting with pain located in the epigastric region, to rule out acute pancreatitis. This has been done here. In this patient you would also order LFTs, FBC looking for evidence of inflammation. MRCP, ERCP and EUS can be considered if necessary.

How well did you know this?
1
Not at all
2
3
4
5
Perfectly
6
Q

A 78 year old woman has a 3 day history of constant left iliac fossa pain. She has a pyrexia of 38 degrees Celsius and left iliac fossa tenderness and guarding. The CT scan demonstrates an inflamed sigmoid colon with numerous diverticulae

What should be the first investigation?

  1. AXR
  2. Laxatives
  3. CT scan
  4. ECG
  5. Oral antibiotics
  6. NG tube
  7. Endoscopy
  8. Ultrasound scan
  9. Laparotomy
  10. Serum amylase
  11. Diagnostic laparoscopy
  12. PR exam
  13. Serum potassium
A
  1. Oral antibiotics

Symptomatic diverticulitis presents with fever, high WCC and LLQ pain. Risk factors for diverticular disease include low dietary fibre and advanced age. Oral antibiotic therapy and analgesia is indicated. If there is no improvement in 72 hours after oral antibiotics then IV antibiotics are indicated. Make sure you understand the differences in the terms: diverticulosis, diverticulitis and diverticular disease.

How well did you know this?
1
Not at all
2
3
4
5
Perfectly
7
Q

A 33 year old woman has collapsed with severe generalised abdominal pain. She is apyrexial, pulse 140/min BP 90/40. Abdomen is rigid and tender with guarding. She denies being pregnant as she has an intra-uterine device (coil) in situ.

What is the diagnosis?
A.	Cholecystitis
B.	Acute salpingitis
C.	Pyelonephritis
D.	Renal/ureteric colic
E.	Ruptured abdominal aortic aneurysm
F.	Appendicitis
G.	Diverticulitis
H.	Urinary tract infection
I.	Bowel obstruction
J.	Bowel perforation
K.	Pancreatitis
L.	Ruptured ectopic pregnancy
A

L. Ruptured ectopic pregnancy

Ectopic pregnancy usually presents between 6-8 weeks after the last normal menstrual period but it can present later on. The risk increases if the woman has had a previous ectopic, surgery on the tubes, genital infections, smokes or uses an IUD. The classic symptoms and signs are pain, vaginal bleeding and amenorrhoea. If the patient is haemodynamically unstable or there is cervical motion tenderness, this may indicate that a rupture has occured or is imminent. Rupture, which is a complication, can present with shock from blood loss and with unusual patterns of referred pain from the presence of intraperitoneal blood. Urine pregnancy test would be expected to confirm pregnancy. Once the patient is confirmed to be pregnant, a transvaginal USS is used to determine the location of the pregnancy. If an intrauterine gestation is visible on USS regardless of whether it is viable, then the chances of having an ectopic pregnancy are incredibly low. Occasionally, an ectopic pregnancy itself can be seen, either as a ‘doughnut sign’ (adnexal mass separate from two clearly seen ovaries) or ‘ring of fire’ (increased blood flood to the ectopic seen on colour Doppler). A transabdominal ultrasound is less sensitive than a TVUS. Treatment approaches can include expactant, medical (methotrexate) or surgical (salpingectomy, salpingostomy).

How well did you know this?
1
Not at all
2
3
4
5
Perfectly
8
Q

A 28 year old woman has a 24 hour history of severe constant, right loin pain and vomiting. She has had rigors and sweats. Urinalysis reveals blood and protein.

What is the diagnosis?
A.	Cholecystitis
B.	Acute salpingitis
C.	Pyelonephritis
D.	Renal/ureteric colic
E.	Ruptured abdominal aortic aneurysm
F.	Appendicitis
G.	Diverticulitis
H.	Urinary tract infection
I.	Bowel obstruction
J.	Bowel perforation
K.	Pancreatitis
L.	Ruptured ectopic pregnancy
A

C. Pyelonephritis

Acute onset fever with rigors, flank pain, vomiting and positive urine dipstick all point to the diagnosis of acute pyelonephritis. Urinalysis is highly sensitive but not very specific. Note that pregnancy is a risk factor for complicated disease as the enlarging uterus compresses the ureters and hormonal changes increase the likelihood of obstructive uropathy. In uncomplicated pyelonephritis, the most common cause is E. coli and gram stain will typically reveal gram negative rods, either E. coli, Proteus or Klebsiella. Gram positive cocci that could be implicated include enterococci and staphylococci. Older patients can often also present non-specifically. Treatment should start before culture results are received to prevent the patient from deteriorating, with empirical antibiotics.

How well did you know this?
1
Not at all
2
3
4
5
Perfectly
9
Q

A 20 year old man has a 24 hour history of abdominal pain, which started in the para-umbilical region, but seems to have moved to his right iliac fossa. He is tender in this area with guarding and rebound tenderness.

What is the diagnosis?
A.	Cholecystitis
B.	Acute salpingitis
C.	Pyelonephritis
D.	Renal/ureteric colic
E.	Ruptured abdominal aortic aneurysm
F.	Appendicitis
G.	Diverticulitis
H.	Urinary tract infection
I.	Bowel obstruction
J.	Bowel perforation
K.	Pancreatitis
L.	Ruptured ectopic pregnancy
A

F. Appendicitis

This is acute inflammation of the appendix. The presentation here is typical of this condition with acute abdominal pain starting in the mid-abdomen and then later localising to the RLQ. This condition is associated with fever, anorexia, N&V and raised neutrophil count. The diagnosis is clinical, though if required, CT scan and USS may show dilation of the appendix outder diameter to >6mm. Definitive treatment here is surgical with the removal of the offending appendix. Note that Rovsing’s sign may be present where pressing on the left side of the abdominal cavity elicits pain in the RLQ. Psoas sign and obturator sign may also be present. These are all rare but do crop up in EMQs. It has been postulated that the higher incidence of this condition in Western society may be attributed to the living conditions and improved personal hygiene.

How well did you know this?
1
Not at all
2
3
4
5
Perfectly
10
Q

A 60 year old man who has had a previous laparotomy for a perforated duodenal ulcer has a 24 hour history of colicky abdominal pain, absolute constipation and vomiting. He had a distended resonant abdomen and high pitched bowel sounds.

What is the diagnosis?
A.	Cholecystitis
B.	Acute salpingitis
C.	Pyelonephritis
D.	Renal/ureteric colic
E.	Ruptured abdominal aortic aneurysm
F.	Appendicitis
G.	Diverticulitis
H.	Urinary tract infection
I.	Bowel obstruction
J.	Bowel perforation
K.	Pancreatitis
L.	Ruptured ectopic pregnancy
A

I. Bowel obstruction

Bowel obstruction has several causes such as adhesions or cancer in the older patient. The cause here is most likely adhesions due to the previous laparotomy this patient has had. The vomiting, distended and painful abdomen and tinkling high pitched bowel signs are all indicative. The proximal segment of bowel dilates and distal bowel collapses. Completely obstructed patients generally require surgery. If, on AXR, air is seen to be seeping past the obstruction then the obstruction is partial. As a standard, all patients should be made NBM and given supplemental oxygen, IV fluids and NG decompression (to reduce flow/gastric contents/air towards the obstruction), unless they are rushed off for an emergency laparotomy because, for example, they have complete obstruction and are peritonitic.

How well did you know this?
1
Not at all
2
3
4
5
Perfectly
11
Q

A 35 year old woman has a 2 day history of severe abdominal pain and profuse vomiting. She has previously had episodes of RUQ pain, particularly after fatty meals. She is jaundiced and mildly tender in her epigastrum. Pulse 120pbm and BP 90/50.

What is the diagnosis?
A.	Cholecystitis
B.	Acute salpingitis
C.	Pyelonephritis
D.	Renal/ureteric colic
E.	Ruptured abdominal aortic aneurysm
F.	Appendicitis
G.	Diverticulitis
H.	Urinary tract infection
I.	Bowel obstruction
J.	Bowel perforation
K.	Pancreatitis
L.	Ruptured ectopic pregnancy
A

A. Cholecystitis

Cholecystitis is acute GB inflammation caused by an obstruction at the cystic duct. It occurs as a major complication of gallstones and classically presents with RUQ pain and fever. There previous episodes of biliary pain here which is a clue. Around 50% of patients who have had one episode of biliary pain will have another within one year. Vomiting is a non-specific symptom which is commonly associated with biliary disease. Mild jaundice is also present in 10% of patients with the condition. This is due to inflammation and oedemaGallstones in EMQs classically involves the Fs (Fat, Forty, Female, Fertile, Fair). USS is the definitive initial investigation. HIDA scanning and MRI may help if the diagnosis remains unclear. Treatment is with cholecystectomy.

How well did you know this?
1
Not at all
2
3
4
5
Perfectly
12
Q

A 70 year old man, who has a long history of hypertension and a recent history of intermittent back pain, collapses with severe central abdominal pain. The pain radiates to his back.

What is the diagnosis?
A.	Cholecystitis
B.	Acute salpingitis
C.	Pyelonephritis
D.	Renal/ureteric colic
E.	Ruptured abdominal aortic aneurysm
F.	Appendicitis
G.	Diverticulitis
H.	Urinary tract infection
I.	Bowel obstruction
J.	Bowel perforation
K.	Pancreatitis
L.	Ruptured ectopic pregnancy
A

E. Ruptured abdominal aortic aneurysm

This is a history of a ruptured AAA. Hypertension is a weak risk factor here. Strong risk factors include male sex (prevalence), female sex (rupture), connective tissue disorders, increased age, FH and cigarette smoking. Males are 4-6 times more likely to have a AAA but female sex increases the risk of rupture. There is the classic abdominal pain radiating to the back here. Examination would likely reveal low BP and compensatory tachycardia due to the blood loss. This has resulted in this patient’s collapse. As this AAA has ruptured, this man will need urgent surgical repair, with standard resuscitation measures. The airway will needed to be managed with supplemental oxygen and ET intubation, a central venous catheter will need to be inserted, an arterial catheter and urinary catheter will also be needed for monitoring, and the target systolic BP is 50-70. Infusing too many fluids may increase the risk of death. The most effective form of surgical repair is an EVAR (endovascular AAA repair), anatomy permitting, otherwise traditional open repair is performed. Open repair has a mortality of 48%. Antibiotics will also be needed to cover bacteria to prevent graft infection. This will be prescribed in line with local protocols.

How well did you know this?
1
Not at all
2
3
4
5
Perfectly
13
Q

A 43 year old housewife complains of a colicky pain in the right upper quadrant which radiates to the back. Associated symptoms are nausea and vomiting. She says it is brought on by fatty foods and not relieved by pain killers bought over the counter. She is also jaundiced.

What is the diagnosis?
A.	Myocardial infarction
B.	Ectopic pregnancy
C.	Bowel obstruction
D.	Hepatitis
E.	Appendicitis
F.	Pancreatitis
G.	Urinary tract infection
H.	Diverticulitis
I.	Colorectal carcinoma
J.	Gastric ulcer
K.	Biliary colic
A

K. Biliary colic

This is cholelithiasis, or gallstones, which is causing biliary colic. The symptoms here are due to the obstruction of the cystic or bile duct, or the erosion of gallstones through the gallbladder. The main causes are defects in cholesterol metabolism and gallbladder hypomotility. Common risk risk factors include older age, female sex, obesity, rapid weight loss, drugs and pregnancy. Jaundice is uncommon in this condition but can develop in patients with choledocholithiasis, which may or may not occur on a background of cholangitis. Nausea often accompanies biliary pain. The pain is also classically worse after food, especially fatty foods. Fever, if present, would suggest a complication such as cholecystitis, cholangitis or pancreatitis. The definitive radiographic test for symptomatic patients is to do an abdominal ultrasound scan (and a CT follow up if unremarkable). MRCP is also an excellent non-invasive imaging method with a high sensitivty and specificity. The definitive treatment is with laparoscopic cholecystectomy. Complications of gallstones include cholecystitis, cholangitis and pancreatitis.

How well did you know this?
1
Not at all
2
3
4
5
Perfectly
14
Q

An 89 year old retired lady presents with generalised abdominal pain, nausea and vomiting of a week’s history. On questioning she hasn’t opened her bowels and there has been no flatus either.

What is the diagnosis?
A.	Myocardial infarction
B.	Ectopic pregnancy
C.	Bowel obstruction
D.	Hepatitis
E.	Appendicitis
F.	Pancreatitis
G.	Urinary tract infection
H.	Diverticulitis
I.	Colorectal carcinoma
J.	Gastric ulcer
K.	Biliary colic
A

C. Bowel obstruction

The cause here in this elderly patient may well be colorectal malignancy, which needs to be considered in all patients who present with large bowel obstruction. The symptom onset here is also gradual which is also suggestive of an enlarging lesion. There may also be recent weight loss and PR bleeding. If this patient had persistent tachycardia, fever and/or abdominal pain and tenderness then you should suspect possible perforation. An urgent surgical opinion is needed in this patient. An erect CXR needs to be ordered initially to assess for intestinal perforation. Diagnosis of obstruction is made on plain AXR. Where doubts exist on XR then contrast enema or CT scan may be useful. The latter may also show the underlying cause such as malignancy, colonic volvulus, stricture or diverticulitis. Note that colonic volvulus tends to be abrupt in onset compared to the other causes where symptom onset tends to be gradual. N&V may be a late sign in large bowel obstruction.

How well did you know this?
1
Not at all
2
3
4
5
Perfectly
15
Q

A young man was rushed to A&E with right iliac fossa pain. It started in the umbilical region. He says he’s also constipated and on examination is tender on light palpation with guarding.

What is the diagnosis?
A.	Myocardial infarction
B.	Ectopic pregnancy
C.	Bowel obstruction
D.	Hepatitis
E.	Appendicitis
F.	Pancreatitis
G.	Urinary tract infection
H.	Diverticulitis
I.	Colorectal carcinoma
J.	Gastric ulcer
K.	Biliary colic
A

E. Appendicitis

This is a classical presentation of this condition. This is acute inflammation of the appendix. Typically there is acute abdominal pain starting in the mid-abdomen and then later localising to the RLQ. This condition is associated with fever, anorexia, N&V and raised neutrophil count. The diagnosis is clinical, though if required, CT scan and USS may show dilation of the appendix outder diameter to >6mm. Definitive treatment here is surgical with the removal of the offending appendix. Note that Rovsing’s sign may be present where pressing on the left side of the abdominal cavity elicits pain in the RLQ. Psoas sign and obturator sign may also be present. These are all rare but do crop up in EMQs. It has been postulated that the higher incidence of this condition in Western society may be attributed to the living conditions and improved personal hygiene.

How well did you know this?
1
Not at all
2
3
4
5
Perfectly
16
Q

An elderly man comes into hospital with abdominal pain and weight loss. Further questioning reveals he has had melaena and altered bowel habit.

What is the diagnosis?
A.	Myocardial infarction
B.	Ectopic pregnancy
C.	Bowel obstruction
D.	Hepatitis
E.	Appendicitis
F.	Pancreatitis
G.	Urinary tract infection
H.	Diverticulitis
I.	Colorectal carcinoma
J.	Gastric ulcer
K.	Biliary colic
A

I. Colorectal carcinoma

The symptoms of colorectal cancer are non-specific but the GI symptoms here combined with weight loss should make you suspicious. Symptoms include change in bowel habit, rectal bleeding and anaemia, commonly associated with right sided cancer. Increased frequency or looser stools, especially when combined with rectal bleeding is common in left sided cancers. Distension, weight loss and vomiting are usually associated with advanced disease. Colonoscopy will be needed in this patient for diagnosis and a biopsy of any suspicious lesion found. Main curative treatment is surgical resection. CEA is the classic tumour marker, which is raised in about 80% but is not really sensitive or specific enough to be useful in diagnosis or screening, but is used to monitor treatment and recurrence in those who have already been diagnosed. Key risk factors for this condition include advanced age, APC mutation, IBD, obesity and Lynch syndrome (HNPCC).

How well did you know this?
1
Not at all
2
3
4
5
Perfectly
17
Q

A 30 year old banker came in with pain in the epigastric region which radiated to his back. He says he can’t keep anything down & sitting forward helps. He is tachycardic, feverish, jaundiced, is in shock and has a rigid abdomen.

What is the diagnosis?
A.	Myocardial infarction
B.	Ectopic pregnancy
C.	Bowel obstruction
D.	Hepatitis
E.	Appendicitis
F.	Pancreatitis
G.	Urinary tract infection
H.	Diverticulitis
I.	Colorectal carcinoma
J.	Gastric ulcer
K.	Biliary colic
A

F. Pancreatitis

This patient has acute pancreatitis and is peritonitic. He has vomited (can’t keep anything down) and is describing mid-epigastric pain radiating around to the back, which in itself is almost diagnostic. Tachycardia here is as a result of hypovolaemia. Jaundice here is suggestive of gallstone aetiology with obstruction to the common bile duct, though pancreatic oedema can itself cause jaundice. Nausea and vomiting is not uncommon and can occur with agitation and confusion. Complicated haemorrhagic pancreatitis may exhibit Cullen’s sign, Grey-Turner’s sign and Fox’s sign. Make sure you know what these are and you are familiar with the causes of acute pancreatitis (GET SMASHED). Those caused by hypocalcaemia may also display Chvostek’s sign and Trousseau’s sign.

How well did you know this?
1
Not at all
2
3
4
5
Perfectly
18
Q

An 18 year old man presents with severe right iliac fossa pain which has been present for about 24 hours. He thinks the pain may have started more centrally. He feels nauseous and has vomited several times. He has a fever and his heart rate is 110bpm

What is the diagnosis?
A.	Gall stones
B.	Aortic aneurysm
C.	Sickle cell crisis
D.	Acute pancreatitis
E.	Typhoid
F.	Peptic ulcer
G.	Myocardial infarction
H.	Acute appendicitis
I.	Ruptured ectopic pregnancy
J.	Ureteric colic
A

D. Acute appendicitis

This is acute inflammation of the appendix. The presentation here is typical of this condition with acute abdominal pain starting in the mid-abdomen and then later localising to the RLQ. This condition is associated with fever, anorexia, N&V and raised neutrophil count. The diagnosis is clinical, though if required, CT scan and USS may show dilation of the appendix outder diameter to >6mm. Definitive treatment here is surgical with the removal of the offending appendix. Note that Rovsing’s sign may be present where pressing on the left side of the abdominal cavity elicits pain in the RLQ. Psoas sign and obturator sign may also be present. These are all rare but do crop up in EMQs. It has been postulated that the higher incidence of this condition in Western society may be attributed to the living conditions and improved personal hygiene.

How well did you know this?
1
Not at all
2
3
4
5
Perfectly
19
Q

A 42 year old homeless man presents to A&E with left-sided abdominal pain which radiates to his back. He appears jaundiced and smells of alcohol.

What is the diagnosis?
A.	Gall stones
B.	Aortic aneurysm
C.	Sickle cell crisis
D.	Acute pancreatitis
E.	Typhoid
F.	Peptic ulcer
G.	Myocardial infarction
H.	Acute appendicitis
I.	Ruptured ectopic pregnancy
J.	Ureteric colic
A

D. Acute pancreatitis

The cardinal symptom of acute pancreatitis is a steady, sudden onset abdominal pain which radiates to the back. It is associated with N&V and a history of either gallstones or alcohol intake is often present (these two are by far the main causative factors). Typical signs are epigastric tenderness, fever and tachycardia. Essential for diagnosis are elevated amylase and/or lipase levels. Initial treatment of this man will involve resuscitation with IV fluids and correcting any electrolyte abnormalities, analgesia and glucose control. The main aim is to prevent complications such as renal failure. This patient needs to stop drinking and sort out his life or else he is at risk of developing chronic pancreatitis. Chronic pancreatitis occurs due to recurrent attacks of acute pancreatitis which leads to chronic scarring. The classic characteristics of chronic pancreatitis are glucose intolerance, pancreatic insufficiency and calcifications.

How well did you know this?
1
Not at all
2
3
4
5
Perfectly
20
Q

A 49 year old man presents with severe right abdominal pain which radiates to the groin. His english is very poor and he is unable to give a good history. He appears very anxious and in a great deal of pain. You notice that he can not lie still, seemingly because of the pain. An abdominal X-ray is entirely normal

What is the diagnosis?
A.	Gall stones
B.	Aortic aneurysm
C.	Sickle cell crisis
D.	Acute pancreatitis
E.	Typhoid
F.	Peptic ulcer
G.	Myocardial infarction
H.	Acute appendicitis
I.	Ruptured ectopic pregnancy
J.	Ureteric colic
A

J. Ureteric colic

This patient has right sided renal colic which classically presents with severe flank pain radiating to the groin. Microscopic haematuria is present in up to 90% of cases. Up to 85% of stones are visible on a plain KUB although urate stones are radiolucent so an unremarkable AXR does not exclude this diagnosis. If the stone is radio-opaque, calcification will be seen within the urinary tract. In pregnancy, a renal USS is first line. The IVP has now been replaced by the CT scan which is the new diagnostic standard. A non-contrast helical (or spiral) CT is preferred due to high sensitivity and specificity and acurately determines presence, site and size of stones. Stones are analysed after they are extracted or when they are expelled to check their composition. It is worth noting that in all females of child bearing age, a urine pregnancy test is necessary to exclude an ectopic pregnancy.

How well did you know this?
1
Not at all
2
3
4
5
Perfectly
21
Q

A 19 year old student presents with abdominal pains and a high fever which has been increasing over the past week. Paracetamol does not help. She has just returned from her gap year backpacking across India. She also suffering from a cough. Examination reveals splenomegaly.

What is the diagnosis?
A.	Gall stones
B.	Aortic aneurysm
C.	Sickle cell crisis
D.	Acute pancreatitis
E.	Typhoid
F.	Peptic ulcer
G.	Myocardial infarction
H.	Acute appendicitis
I.	Ruptured ectopic pregnancy
J.	Ureteric colic
A

E. Typhoid

Typhoid (or enteric fever) is a faecal-oral illness caused by Salmonella enterica, serotype S typhi, Salmonella enterica and S paratyphi. There are over 2500 serovars for S enterica. In this country, it is mainly from people who have returned from a country where it is endemic – India, for example, which has the highest incidence of this disease. Mexico should also ring alarm bells. The water supplies are not treated and sanitary conditions are dire which prompts transmission of this infection. Humans are the only known reservoir. The vaccine only offers moderate protection and does not protect against paratyphoid infection.

This person has a high fever which is a hallmark of infection (sometimes in a step-wise manner) which is not responding to paracetamol (it persists) and blood culture (you need a big sample of blood for testing as count is usually low) here may show a gram negative bacilli. Hepatomegaly and splenomegaly may exist in this condition. The fever of typhoid classically increases incrementally until a persistent fever with temperature 39-41 is established. There are also typically flu like symptoms after onset of fever and cough. Remember that characteristic findings such as bradycardia and rose spots may not be present and indeed rose spots may not be easy to spot in those with dark skin. Rose spots are blanching red lesions reported in 5-30% of cases usually occuring on the chest or abdomen.

This patient needs antibiotics though the temperature will fall over about week. A third generation cephalosporin is indicated due to resistance to fluoroquinolones in the Indian sub-continent. If the sensitivity panel returns and shows that this organism is sensitive to all antibiotics then ciprofloxacin should be given.

How well did you know this?
1
Not at all
2
3
4
5
Perfectly
22
Q

A 68 year old obese Asian man presents with severe upper abdominal pain and nausea. He is sweaty and says he cannot catch his breath. The abdomen looks normal and there are no palpable masses. A full blood count is ordered but the results aren’t back yet.

What is the diagnosis?
A.	Gall stones
B.	Aortic aneurysm
C.	Sickle cell crisis
D.	Acute pancreatitis
E.	Typhoid
F.	Peptic ulcer
G.	Myocardial infarction
H.	Acute appendicitis
I.	Ruptured ectopic pregnancy
J.	Ureteric colic
A

G. Myocardial infarction

MI can present with an atypical location of pain, which may be in the epigastrium. The rest of the features here point towards MI. The nature of the pain may also be atypical in real life and may even be burning, throbbing or the patient may just describe feeling uneasy. However, chest pain is classically severe and heavy in nature, located centrally with possible radiation to the left arm or jaw and lasts for >20 minutes. This is however an important diagnosis not to miss even if the patient presents atypically.

Risk factors incorporate the standard set of cardiovascular risks such as smoking, high BP, DM, obesity and dyslipidaemia. If ECG shows STEMI, new LBBB or confirmed posterior MI then PCI/thrombolysis is indicated. It is worth noting that RV infarction is present in 40% of inferior infarcts so if ST elevation is seen in II, III and aVF, right sided ECG leads should be obtained. Cardiac biomarkers include CK-MB and troponin.

How well did you know this?
1
Not at all
2
3
4
5
Perfectly
23
Q

A 79 year old man who suffers from constipation presents with sudden onset colicky pain in the left iliac fossa. He has not passed stool or flatus for several days. He is generally well and has no history of weight loss. AXR confirms large bowel obstruction.

What is the diagnosis?
A.	Mesenteric embolus
B.	Colon carcinoma
C.	Inguinal hernia
D.	Meckel's diverticulum
E.	Adhesions
F.	Distal intestinal obstruction syndrome
G.	Sigmoid volvulus
H.	Meconium ileus
A

G. Sigmoid volvulus

There is no history of surgery here to suggest adhesions. Malignancy would cause a gradual symptom onset. This is also unlikely to be a hernia when compared to a colonic volvulus – especially when constipation is a risk factor. Symptoms in colonic volvulus are abrupt in onset. A volvulus is bowel obstruction occuring due to a loop of bowel twisting on its own mesenteric axis. Broadly speaking, there are three types: small bowel, sigmoid and gastric. A sigmoid volvulus is something you need to be able to recognise on AXR and it appears as a dilated loop of large bowel present in the lower abdomen, resembling a coffee bean shape (or like an upside down U shape). The rest of the bowel is usually dilated. For a caecal volvulus, the caecum leaves the RLQ to appear like a second satomach bubble in the centre of the film. There is often associated small bowel dilation. A gastric volvulus is very rare. Note that in Africa, sigmoid volvulus accounts for up to 50% of cases of bowel obstruction.

How well did you know this?
1
Not at all
2
3
4
5
Perfectly
24
Q

A 50 year old woman has lost weight over the last few months and has noticed that she has become more constipated than usual. She presents with abdominal pain and distension and has not passed stool for days.

What is the diagnosis?
A.	Mesenteric embolus
B.	Colon carcinoma
C.	Inguinal hernia
D.	Meckel's diverticulum
E.	Adhesions
F.	Distal intestinal obstruction syndrome
G.	Sigmoid volvulus
H.	Meconium ileus
A

B. Colon carcinoma

The symptom onset is more gradual here and this excludes a volvulus. The weight loss and GI symptoms in this lady point to colorectal carcinoma as the cause for colonic obstruction. This is more common in older people. When considering epidemiology, approximately 90% of cases of large bowel obstruction are caused by underlying colorectal cancer, 5% by colonic volvulus and 3% from a benign stricture (for example, diverticular, inflammatory, ischaemic etc). The remaining 2% are accounted for by rare conditions such as hernia and endometriosis.

How well did you know this?
1
Not at all
2
3
4
5
Perfectly
25
Q

A thin 24 year old gentleman has suffered from frequent chest infections since childhood. Last week he had a cough, productive of lots of thick sputum. He comes to the A&E department with abdominal pain and distension and is not passing any stool.

What is the diagnosis?
A.	Mesenteric embolus
B.	Colon carcinoma
C.	Inguinal hernia
D.	Meckel's diverticulum
E.	Adhesions
F.	Distal intestinal obstruction syndrome
G.	Sigmoid volvulus
H.	Meconium ileus
A

F. Distal intestinal obstruction syndrome

his is distal intestinal obstruction syndrome, which was previously known as meconium ileus equivalent. Here, abnormal salt and water balance in the intestines can lead to thickened and congealed stool and intestinal mucus which leads to bowel obstruction, usually occuring in the terminal ileum. This is known as meconium ileus in the neonate, and distal intestinal obstruction syndrome after the neonatal period. This condition occurs in patients with cystic fibrosis, which this patient’s history points towards. The first line treatment is with water soluble contrast enema and oral osmotic agents. The patient should be made NBM and an NG tube may also be placed for drainage. If medical management fails then surgery is indicated to resolve the obstruction.

Remember that meconium is the sticky tarry material found in the intestines of newborns and is composed of bile salts, amniotic fluid and substances ingested in utero. It is normally completely evacuated in the first few days following birth due to the trypsin and other pancreatic enzymes. However, in CF, as pancreatic exocrine ducts become blocked, the sticky meconium becomes inspissated and blocks the lower ileum of the newborn leading to obstruction. Furthermore, in some CF patients such as this one, a related disorder, distal intestinal obstruction syndrome, occurs in later life as a result of chronic constipation and malabsorption.

How well did you know this?
1
Not at all
2
3
4
5
Perfectly
26
Q

A 35 year old builder was lifting some heavy loads. That evening he noticed a lump in his right groin and after some hours it became painful, red and more swollen. He was not able to reduce the lump and by the time he arrives in the A&E department he is tachycardic and pyrexial.

What is the diagnosis?
A.	Mesenteric embolus
B.	Colon carcinoma
C.	Inguinal hernia
D.	Meckel's diverticulum
E.	Adhesions
F.	Distal intestinal obstruction syndrome
G.	Sigmoid volvulus
H.	Meconium ileus
A

C. Inguinal hernia

Builders are involved in heavy lifting which increases intra-abdominal pressure, predisposing to the development of a hernia. This hernia has strangulated and is irreducible. This occurs when the blood supply to the contents of a hernia is cut off due to the hernia twisting upon itself. In this case, a loop of bowel has strangulated, leading to intestinal obstruction. Femoral hernias are more likely to strangulate due to their relatively narrow neck. Urgent surgical intervention is needed in this patient with open repair. Treatment differs depending on whether the strangulated bowel is viable or not.

How well did you know this?
1
Not at all
2
3
4
5
Perfectly
27
Q

An 18 year old female presents with acute onset right iliac fossa pain. After a negative beta-HCG test she is taken to theatre for laparoscopy where the appendix is seen to be normal but there is inflammation of the fallopian tubes.

What is the diagnosis?
A.	Streptococcus pneumoniae
B.	Candida albicans
C.	Hepatitis A
D.	Helicobacter pylori
E.	Neisseria gonorrhoea
F.	E coli
G.	Hepatitis C
H.	Salmonella
A

E. Neisseria gonorrhoea

Pelvic inflammatory disease is an acute ascending infection of the female tract that is often associated with Neisseria gonorrhoeae or Chlamydia trachomatis. Key risk factors include prior infection with chlamydia or gonorrhoea or PID, young age of onset of sexual activity, unprotected sex with multiple partners and IUD use. Signs and symptoms vary and can include tenderness of the lower abdomen, adnexal tenderness and cervical motion tenderness. Fever and cervical or vaginal discharge may also be present. Complications include tubo-ovarian abscess and subsequent infertility or ectopic pregnancy due to scarred or obstructed fallopian tubes.

How well did you know this?
1
Not at all
2
3
4
5
Perfectly
28
Q

A 50 year old gentleman visits his GP complaining of ‘indigestion’ especially if he eats spicy food. He has epigastric pain after eating and has recently lost some weight.

What is the diagnosis?
A.	Streptococcus pneumoniae
B.	Candida albicans
C.	Hepatitis A
D.	Helicobacter pylori
E.	Neisseria gonorrhoea
F.	E coli
G.	Hepatitis C
H.	Salmonella
A

D. Helicobacter pylori

Helicobacter pylori is the appropriate choice for this question. Epigastric pain and tenderness related to eating a meal is typical of a peptic ulcer. The pain is generally relieved by drinking milk. 80% are duodenal and 20% are gastric. Ulcers may cause iron deficiency anaemia and associated symptoms may feature. Key risk factors are NSAID use, H. pylori infection, smoking and a family history of PUD. Zollinger-Ellison syndrome should be considered if there are multiple ulcers or ulcers refractory to treatment. Management is aimed at correcting the underlying cause such as discontinuing NSAIDs. Testing for Helicobacter pylori can be with breath testing with radiolabelled urea or stool antigen testing. H. pylori eradication should be started if the organism is present with triple therapy. Otherwise, a PPI is indicated.

Helicobacter pylori is known to have a role in the aetiology of this condition. If we exclude NSAIDs then 90% of duodenal ulcers and more than 70% of gastric ulcers have Helicobacter pylori infection compared to 30% and 50% in the general population. Infection increases the lifetime risk of getting a peptic ulcer. The likely postulated mechanism is through gastrin and acid hypersecretion (duodenal ulcers) and local mucosal damage (gastric ulcers).

Gastric ulcers classically cause pain which is exacerbated by eating and immediately relieved on vomiting. There is usually also weight loss due to a fear of food and its association with pain. Duodenal ulcers are classically made worse by hunger and are relieved by eating and the patient may wake at night with the pain. As a result, weight gain is typically a feature. In reality, it is difficult to differentiate the site of the ulcer based on these features.

How well did you know this?
1
Not at all
2
3
4
5
Perfectly
29
Q

A 70 year old man comes to the A&E department with a fever and productive cough for the previous week. He also complains of pain in the right upper quadrant. On examination the abdomen is soft and there is no organomegaly.

What is the diagnosis?
A.	Streptococcus pneumoniae
B.	Candida albicans
C.	Hepatitis A
D.	Helicobacter pylori
E.	Neisseria gonorrhoea
F.	E coli
G.	Hepatitis C
H.	Salmonella
A

A. Streptococcus pneumoniae

The pain in the RUQ here is due to pneumonia of the right lower pulmonary zone. Respiratory symptoms are present here with a productive cough and the patient is feverish. Additionally abdominal examination is unremarkable in every way. A CXR will be needed here as it is the most specific and sensitive test for pneumonia. Remember that sometimes patients with lower lobe pneumonia may present with pain the abdomen.

How well did you know this?
1
Not at all
2
3
4
5
Perfectly
30
Q

A 42 year old intravenous drug user is admitted with jaundice, abdominal distension and right upper quadrant pain. On examination he is found to have a fullness in the right upper quadrant and a fluid thrill in the abdomen.

What is the diagnosis?
A.	Streptococcus pneumoniae
B.	Candida albicans
C.	Hepatitis A
D.	Helicobacter pylori
E.	Neisseria gonorrhoea
F.	E coli
G.	Hepatitis C
H.	Salmonella
A

G. Hepatitis C

Worldwide, HBV and HCV are also major causes of cirrhosis. This patient has decompensated chronic liver disease with ascites and jaundice. The most common route which HCV is transmitted is through illicit IVDU. Following the acute exposure, most patients (55-85%) will go on to develop chronic hepatitis C. Most infections are asymptomatic. Treatment involves, usually, pegylated interferon and ribavirin with the aim of getting rid of viraemia. Triple therapy is used for the most common genotype 1, and this consists of the above two drugs as well as a HCV protease inhibitor. Long term complications of infection include cirrhosis or HCC. This patient has developed the first complication.

There are classic signs of liver disease with ascites as a sign of decompensation. It is really important to note that HCV is not sexually transmitted. This may sound like a paradox… but HCV is trasmitted by blood, not sex. The reason homosexual intercourse is a risk factor for HCV and HCV is increasing in incidence among homosexual men in London is likely due to traumatic anal intercourse. Hence, the virus is still transmitted by blood. This is what the majority of studies seem to suggest, although there is still a bit of contention here, as there is with whether HIV can be transmitted through oral sex.

How well did you know this?
1
Not at all
2
3
4
5
Perfectly
31
Q

A 70 year old smoker who has a 6 week history of epigastric discomfort worse on eating. He has lost his appetite as he has a sense of fullness all the time & has lost 3 kilos in weigh. Abdominal examination is unremarkable.

Which single investigation is the most appropriate to confirm the likely diagnosis in the patients described?
A.	Serum Amylase
B.	H. pylori antibodies
C.	Colonoscopy
D.	Stool examination for pathogens
E.	Full blood count, ESR, creatinine, electrolytes & liver function tests
F.	Barium meal
G.	Ultrasound abdomen
H.	History only
I.	Barium enema
J.	Gastroscopy
A

J. Gastroscopy

Weight loss is one of the most common presenting symptoms in patients with gastric cancer. Epigastric pain is present in about 80% and may resemble that of a gastric ulcer. Although commonly mentioned in EMQs, lymphadenopathy is an uncommon presentation. On examination there may be the presence of a left supraclavicular node (Virchow’s node), a periumbilical nodule (Sister Mary Joseph’s nodule) or a left axillary nodule (Irish node). These are rare findings. A mass may be palpable in advanced disease. More proximal tumours can present with dysphagia. Acanthosis nigricans, a black velvety rash, may also be present.

Strong risk factors include pernicious anaemia, Helicobacter pylori and the consumption of N-nitroso compounds found in cured meats. The peak incidence occurs between 50-70 and men are twice as likely to have gastric cancer. Most are adenocarcinomas. The first test to order for suspected gastric malignancy is an urgent upper GI endoscopy with biopsy of the lesion. The mainstay of treatment is surgical resection unless there is evidence of metastatic disease.

How well did you know this?
1
Not at all
2
3
4
5
Perfectly
32
Q

A 45 year old unmarried labourer who increasingly severe central abdominal pain over the last 3 hours. The pain radiates through to his back & makes him vomit. He has no previous history. On examination is cold & sweaty, pulse 120, BP 90/70, & has guarding over his whole abdomen.

Which single investigation is the most appropriate to confirm the likely diagnosis in the patients described?
A.	Serum Amylase
B.	H. pylori antibodies
C.	Colonoscopy
D.	Stool examination for pathogens
E.	Full blood count, ESR, creatinine, electrolytes & liver function tests
F.	Barium meal
G.	Ultrasound abdomen
H.	History only
I.	Barium enema
J.	Gastroscopy
A

A. Serum amylase

This patient has acute pancreatitis. He has vomited and is describing mid-epigastric pain radiating around to the back. This pain is classically relieved if the patient curls up and is worse with movement. This patient also vomited too. Complicated haemorrhagic pancreatitis may exhibit Cullen’s sign, Grey-Turner’s sign and Fox’s sign. Make sure you know what these are and you are familiar with the causes of acute pancreatitis. Those caused by hypocalcaemia may display Chvostek’s sign and Trousseau’s sign.

Key to diagnosis is serum amylase or lipase levels which are massively elevated. Prognostic criteria are outlined in Ranson’s criteria applied on admission and after 48 hours, or the modified Glasgow score which you can find in your Oxford Handbook. An abdominal CT is however the most sensitive and specific study and findings may include enlargement of the pancreas with irregular contours, necrosis, pseudocysts and peripancreatic fat obliteration.

For interest, urinary trypsinogen-2 is now considered a better screening test than amylase but is not currently clinically used.

How well did you know this?
1
Not at all
2
3
4
5
Perfectly
33
Q

A 20 year old secretary who complains of abdominal discomfort & bloating over the last 3 months. She also has intermittent diarrhoea, but when she opens her bowels, her symptoms are relieved. There is no blood or mucus in her stools. Abdominal & rectal examination is normal.

Which single investigation is the most appropriate to confirm the likely diagnosis in the patients described?
A.	Serum Amylase
B.	H. pylori antibodies
C.	Colonoscopy
D.	Stool examination for pathogens
E.	Full blood count, ESR, creatinine, electrolytes & liver function tests
F.	Barium meal
G.	Ultrasound abdomen
H.	History only
I.	Barium enema
J.	Gastroscopy
A

H. History only

The intermittent diarrhoea, dyspepsia and bloating without symptoms suggestive of IBD make IBS a more likely diagnosis. IBS is a chronic condition with abdominal pain associated with bowel dysfunction and is a diagnosis of exclusion. The pain or discomfort may be relieved by defecation, as in this case. Examination is usually unremarkable and the diagnosis is based on the patient’s history in line with the Rome Criteria. If the patient presents with any worrying symptoms, then these will warrant a more thorough investigation. Treatment depends on the patient’s predominant symptoms. Antispasmodics relieve abdominal pain or discomfort but do not affect bowel habit. Examples include peppermint oil and dicycloverine. Laxatives can also be used such as lactulose. Lifestyle and dietary modifications combined with reassurance remain the 1st line intervention for functional bowel disease. IBS is linked with stressful jobs such as working as a secretary and there is a female/male ratio of 2:1.

How well did you know this?
1
Not at all
2
3
4
5
Perfectly
34
Q

A 50 year old housewife & mother of 5 has sudden severe epigastric pain that radiates to the back on the right & has vomited. She puts it down to her recent meal of fish & chips, as she usually never eats fatty food. Examination is somewhat difficult as she is obese but you think she has some guarding over the epigastium and right hypochondrium.

Which single investigation is the most appropriate to confirm the likely diagnosis in the patients described?
A.	Serum Amylase
B.	H. pylori antibodies
C.	Colonoscopy
D.	Stool examination for pathogens
E.	Full blood count, ESR, creatinine, electrolytes & liver function tests
F.	Barium meal
G.	Ultrasound abdomen
H.	History only
I.	Barium enema
J.	Gastroscopy
A

G. Ultrasound abdomen

Cholecystitis is acute GB inflammation caused by an obstruction at the cystic duct. It occurs as a major complication of gallstones and classically presents with RUQ pain and fever. Gallstones in EMQs classically involves the Fs (Fat, Forty, Female, Fertile, Fair). USS is the definitive initial investigation. HIDA scanning and MRI may help if the diagnosis remains unclear. Treatment is with cholecystectomy. Make sure you know the difference between ascending cholangitis, cholecystitis and biliary colic.

How well did you know this?
1
Not at all
2
3
4
5
Perfectly
35
Q

A 25 year old previously well man who has a 3 day history of abdominal cramps diarrhoea with bloody stools 5 or 6 times a day. Examination shows a soft but tender abdomen.

Which single investigation is the most appropriate to confirm the likely diagnosis in the patients described?
A.	Serum Amylase
B.	H. pylori antibodies
C.	Colonoscopy
D.	Stool examination for pathogens
E.	Full blood count, ESR, creatinine, electrolytes & liver function tests
F.	Barium meal
G.	Ultrasound abdomen
H.	History only
I.	Barium enema
J.	Gastroscopy
A

D. Stool examination for pathogens

This patient has most likely has infectious diarrhoea. There is a short history in a previous well person. This man has probably eaten something dodgy like a kebab. Do you know the organisms that can cause bloody diarrhoea? Think of the organisms which cause bloody diarrhoea such as EHEC. The mainstay of treatment is rehydration and supportive therapy. Antibiotics may be indicated, particularly in severe cases. The diagnosis would be confirmed by stool examination.

How well did you know this?
1
Not at all
2
3
4
5
Perfectly
36
Q

An 82 year old female presents with a large lump which appears at the anus after defecation, & spontaneously on coughing. The lump is uncomfortable & the patient has tenesmus. She also has incontinence & has noticed mucus PR.

For each question choose the most appropriate diagnosis from the list of options.
A.	Inflammatory bowel disease
B.	Fistula
C.	Fissure
D.	Abscess
E.	Intussusception
F.	Irritable bowel syndrome
G.	Haemorrhoids
H.	Pilonidal sinus
I.	Prolapse
A

I. Prolapse

Rectal prolapse tends to present as a mass protruding from the rectum, especially with straining on the toilet. This may be associated with mucous or blood-stained discharge, pain and even faecal incontinence. In contrast, haemorrhoids tend to present with painless PR bleeding or perianal pain with a tender mass in the area. This can be confirmed on examination. Tenesmus is the constant feeling of needing to pass stools, even if there is nothing to pass. It may also be a symptom of rectal carcinoma, and is caused by a SOL.

How well did you know this?
1
Not at all
2
3
4
5
Perfectly
37
Q

A 20 year old male presents with painful defecation which persists for 30 mins afterwards. The stool is smeared with blood, & he has noticed recent constipation.

For each question choose the most appropriate diagnosis from the list of options.
A.	Inflammatory bowel disease
B.	Fistula
C.	Fissure
D.	Abscess
E.	Intussusception
F.	Irritable bowel syndrome
G.	Haemorrhoids
H.	Pilonidal sinus
I.	Prolapse
A

C. Fissure

This patient has an anal fissure. This causes severe pain on defecation and may continue for 1 to 2 hours. A small amount of fresh blood is often passed on the stool. Hard stools is a strong risk factor and this patient’s constipation will likely be the cause. Opiates are associated with constipation and subsequently anal fissures too and fissures may also occur in the third trimester of pregnancy or after delivery. Initial treatment is with topical GTN or diltiazem along with supportive measures such a high fibre diet. Resistant or chronic fissures may benefit from surgical measures or botulinum toxin.

How well did you know this?
1
Not at all
2
3
4
5
Perfectly
38
Q

A 50 year old female presents with a purulent discharge from the anal region & has recurrent episodes of pain, which is intense & throbbing. On examination there was pruritis ani.

For each question choose the most appropriate diagnosis from the list of options.
A.	Inflammatory bowel disease
B.	Fistula
C.	Fissure
D.	Abscess
E.	Intussusception
F.	Irritable bowel syndrome
G.	Haemorrhoids
H.	Pilonidal sinus
I.	Prolapse
A

D. Abscess

Typically, this presents with perianal pain. The location is important and affects management and diagnosis. If it is inter-sphincteric, then anaesthesia tends to be required to examine the rectal passage adequately for diagnosis – the pain tends to be so bad that a DRE is impossible otherwise. Abscesses found above levator ani may minic an abdominal condition. CT/MRI may be needed to establish the diagnosis. Risk factors include conditions like Crohn’s and anal fistulae (a complication of an abscess can also be a fistula). This accounts for the patient’s symptoms such as pruritis ani. A pilodinal abscess would be difficult to distinguish from an anorectal abscess on history alone but are usually located in the inter-gluteal region and often have a sinus tract in the midline. If the two are difficult to distinguish (such as if the patient cannot be examined), then MRI or CT of the pelvis may be used. Treatment involves drainage of the abscess surgically and the fistula, if present, can also be managed with a fistulotomy or seton insertion. Some patients will also get adjunctive antibiotics, such as the elderly and diabetics.

How well did you know this?
1
Not at all
2
3
4
5
Perfectly
39
Q

A 40 year old male presents with PR bleeding & a palpable lump from anus, with associated mucus discharge. There is blood splashed around lavatory pan

For each question choose the most appropriate diagnosis from the list of options.
A.	Inflammatory bowel disease
B.	Fistula
C.	Fissure
D.	Abscess
E.	Intussusception
F.	Irritable bowel syndrome
G.	Haemorrhoids
H.	Pilonidal sinus
I.	Prolapse
A

G. Haemorrhoids

Haemorrhoids are vascular rich cushions in the anal canal and presents, typically, as painless bright PR bleeding or with sudden onset pain in the area associated with a palpable mass. Pruritus ani is common and there is often perianal pain or discomfort. Diagnosis is made visually. Grade 1 is limited to within the anal canal. Grade 2 protrudes but spontaneously reduces when the patient stops straining. Grade 3 protrudes and reduces fully on manual pressure. Grade 4 is irreducible. Treatment includes fibre, ligation, photocoagulation, sclerotherapy or surgical haemorrhoidectomy. Haemorrhoidectomy is the treatment of choice of choice for patients with grade 4 haemorrhoids or for any patient who has failed with more conservative treatment such as sclerotherapy.

How well did you know this?
1
Not at all
2
3
4
5
Perfectly
40
Q

A 70 year old male who presents with significant weight loss & progressive painless jaundice. Ultrasound demonstrates a dilated biliary system down to the head of the pancreas.

For each patient below, choose the most likely diagnosis from the list of options.
A.	Carcinoma tail of pancreas
B.	Hepatitis B
C.	Cholecystitis
D.	Cirrhosis
E.	Ascending cholangitis
F.	Autoimmune hepatitis
G.	Carcinoma head of pancreas
H.	Drug induced hepatitis
I.	Sickle cell anaemia
J.	Gallstone in common bile duct
K.	Infectious mononucleosis
L.	Hepatitis A
A

G. Carcinoma head of pancreas

Pancreatic cancer (of the head) typically presents with painless obstructive jaundice and weight loss and generally presents late. Whipple’s procedure or Traverso-Longmire procedure (pancreaticoduodenectomy) offers the only hope of a cure but only a small minority are elegible for these procedures. The first tests to order are an abdominal USS and LFTs. Note Courvoisier’s law: Jaundice and a palpable painless gallbladder is unlikely to be caused by gallstones. The tumour marker for pancreatic cancer is CA19-9 which is useful in preoperative staging.

How well did you know this?
1
Not at all
2
3
4
5
Perfectly
41
Q

A 46 year old sales rep has recently lost his job & presents with jaundice & ascites. Ultrasound of the biliary tract is normal.

For each patient below, choose the most likely diagnosis from the list of options.
A.	Carcinoma tail of pancreas
B.	Hepatitis B
C.	Cholecystitis
D.	Cirrhosis
E.	Ascending cholangitis
F.	Autoimmune hepatitis
G.	Carcinoma head of pancreas
H.	Drug induced hepatitis
I.	Sickle cell anaemia
J.	Gallstone in common bile duct
K.	Infectious mononucleosis
L.	Hepatitis A
A

D. Cirrhosis

This sales rep who has just lost his job has been hitting the bottle. The gynaecomastia is a sign of chronic liver disease and ascites indicates a degree of decompensation. Alcoholic liver disease is the most common cause of cirrhosis in the Western world.

How well did you know this?
1
Not at all
2
3
4
5
Perfectly
42
Q

An obese 40 year old woman, with a history of episodic right upper quadrant pain, presents with rapid onset of jaundice with severe abdominal pains, fever & rigors.

For each patient below, choose the most likely diagnosis from the list of options.
A.	Carcinoma tail of pancreas
B.	Hepatitis B
C.	Cholecystitis
D.	Cirrhosis
E.	Ascending cholangitis
F.	Autoimmune hepatitis
G.	Carcinoma head of pancreas
H.	Drug induced hepatitis
I.	Sickle cell anaemia
J.	Gallstone in common bile duct
K.	Infectious mononucleosis
L.	Hepatitis A
A

E. Ascending cholangitis

Charcot’s triad of ascending cholangitis: fever with or without rigors, RUQ pain and fever. Cholangitis is infection of the biliary tree and can quickly become septic. Drainage of the biliary tree is crucial and is undertaken via ERCP. Make sure you know the difference between ascending cholangitis, cholecystitis and biliary colic.

How well did you know this?
1
Not at all
2
3
4
5
Perfectly
43
Q

A 24 year old unemployed rough sleeping male presents unwell, with jaundice, anorexia & lassitude. He has a number of venepuncture marks in the antecubital fossae.

For each patient below, choose the most likely diagnosis from the list of options.
A.	Carcinoma tail of pancreas
B.	Hepatitis B
C.	Cholecystitis
D.	Cirrhosis
E.	Ascending cholangitis
F.	Autoimmune hepatitis
G.	Carcinoma head of pancreas
H.	Drug induced hepatitis
I.	Sickle cell anaemia
J.	Gallstone in common bile duct
K.	Infectious mononucleosis
L.	Hepatitis A
A

B. Hepatitis B

This could well be HCV although this is not an option. There are HBV risk factors in this patient including IVDU. Serum LFTs will shown a transaminitis. HBsAg, HBcAg and HBeAg can be measured. In HCV, HCV RNA can be measured. HAV is not associated with IVDU. Serum IgM anti-HAV can be measured.

How well did you know this?
1
Not at all
2
3
4
5
Perfectly
44
Q

A 20 year old man has been constipated since childhood. He opens his bowels once or twice a week & has noticed faecal soiling.

For each of the following patients choose the most likely diagnosis from the list of options.
A.	Irritable bowel syndrome
B.	Hypothyroidism
C.	Hirschsprung’s disease
D.	Pelvic trauma
E.	Diverticular disease
F.	Chronic laxative abuse
G.	Colorectal carcinoma
H.	Hypercalcaemia
I.	Parkinson’s disease
J.	Adverse effect of drugs
A

C. Hirschsprung’s disease

In this condition there is colonic obstruction associated with absent intramural ganglion cells. The lumen is hence tonically contracted. Hirschsprung’s disease is commonly diagnosed in the first year of life and presents with vomiting, distension and/or colitis. There tends to be explosure liquid foul stools, delayed meconium passage and fever (enterocolitis). However, sometimes, it can present with intermittent bouts of symptoms later on in life and have minimal or absent symptoms in the first few days/weeks (adult presentations are rare). It can be associated with Down’s and MEN2A.

The diagnosis is definitively made on a rectal biopsy with stain for ganglion cells in the submucosal plexus. This will be absent in Hirschsprung’s with the presence of other features such as thickened non-myelinated nerves and increased acetylcholinesterase. A contrast enema will also be done and will show a contracted distal bowel with the proximal bowel dilated, making the location of the transition zone visible on XR. An AXR is always the first investigation performed but a normal film does not exclude the possibility of this diagnosis (it is a non-specific investigation, though having said that, if there are no distended bowel loops then Hirschsprung’s is unlikely). Manometry is not usually performed, but the reflex where when the rectum is distended, pressure in the anal canal falls (as the internal sphincter relaxes) is absent. Initial treatment is with irrigation followed by surgery.

How well did you know this?
1
Not at all
2
3
4
5
Perfectly
45
Q

A 40 year old woman has constipation, weight gain & menorrhagia. She opens her bowels only twice a week. Pulse is 50/min & she has dry skin.

For each of the following patients choose the most likely diagnosis from the list of options.
A.	Irritable bowel syndrome
B.	Hypothyroidism
C.	Hirschsprung’s disease
D.	Pelvic trauma
E.	Diverticular disease
F.	Chronic laxative abuse
G.	Colorectal carcinoma
H.	Hypercalcaemia
I.	Parkinson’s disease
J.	Adverse effect of drugs
A

B. Hypothyroidism

This patient has hypothyroidism. Worldwide, the most common cause is iodine deficiency. Other causes include Hashimoto’s or secondary and tertiary hypothyroidism. It can also result from viral de Quervain’s thyroiditis or postpartum thyroiditis. Symptoms include those mentioned (depression, fatigue, weight gain, bradycardia and sluggish reflexes) as well as others such as constipation, cold intolerance, menstrual problems in females, dry skin and muscle cramps. Diagnosis is based on measurement of TSH and thyroid hormones. Treatment is by replacement of T4 with or without T3 in combination. If the patient has normal T3 and T4 but mildly elevated TSH, this is described as subclinical hypothyroidism.

How well did you know this?
1
Not at all
2
3
4
5
Perfectly
46
Q

A 35 year old woman has mild long-standing constipation without weight loss. She passes hard pellet-like stools, often with straining & a feeling of incomplete evacuation. Examination is normal.

For each of the following patients choose the most likely diagnosis from the list of options.
A.	Irritable bowel syndrome
B.	Hypothyroidism
C.	Hirschsprung’s disease
D.	Pelvic trauma
E.	Diverticular disease
F.	Chronic laxative abuse
G.	Colorectal carcinoma
H.	Hypercalcaemia
I.	Parkinson’s disease
J.	Adverse effect of drugs
A

A. Irritable bowel syndrome

IBS is a chronic condition with abdominal pain associated with bowel dysfunction and is a diagnosis of exclusion. The pain or discomfort may be relieved by defecation. Examination is usually unremarkable and the diagnosis is based on the patient’s history in line with the Rome Criteria. If the patient presents with any worrying symptoms, then these will warrant a more thorough investigation. Treatment depends on the patient’s predominant symptoms. Antispasmodics relieve abdominal pain or discomfort but do not affect bowel habit. Examples include peppermint oil and dicycloverine. Laxatives can also be used such as lactulose. Lifestyle and dietary modifications combined with reassurance remain the 1st line intervention for functional bowel disease. IBS is linked with stressful jobs and there is a female/male ratio of 2:1.

47
Q

A 50 year old man fell from a ladder & injured his back. He requires regular pain-killers for back pain. Since the accident he has had difficulty opening his bowels & has noticed reduced bowel frequency.

For each of the following patients choose the most likely diagnosis from the list of options.
A.	Irritable bowel syndrome
B.	Hypothyroidism
C.	Hirschsprung’s disease
D.	Pelvic trauma
E.	Diverticular disease
F.	Chronic laxative abuse
G.	Colorectal carcinoma
H.	Hypercalcaemia
I.	Parkinson’s disease
J.	Adverse effect of drugs
A

J. Adverse effect of drugs

This patient is on opiates. Let us review opiates such as morphine and codeine. They are commonly prescribed for analgesia and can also be used as an anti-tussive (suppresses ACh and NK release needed for cough activation and inteferes with serotonin receptors in the DRN). They are also abused for their euphoric effects mediated by their action on dopamine release at the nucleus accumbens. They have a host of side effects, including, in this case, GI effects (receptors present on the myenteric and submucosal plexus) of decreased gastric motility, emptying and increased gut water reabsorption… in short, it causes constipation. Other side effects include respiratory depression (desensitises central chemoreceptors to lessen the response to PaCO2), N&V by stimulating the CTZ, pupillary constriction via the Edinger-Westphal nucleus and causes symptoms like pruritis, urticaria, hypotension through histamine release via a direct effect on mast cells.

Arrestins also promote internalisation of opioid receptors which leads to tolerance with prolonged treatment. Prolonged use will also lead to dependance and withdrawl effects if stopped, such as cravings, and the physical effects may be very profound, resembling flu. Naloxone is indicated in an overdose. It is worth looking at the pathways to help you remember the effects.

48
Q

A 52 year old woman has recently developed constipation & feels that she does not completely empty her rectum on defecation. She has passed blood per rectum on 2 occasions.

For each of the following patients choose the most likely diagnosis from the list of options.
A.	Irritable bowel syndrome
B.	Hypothyroidism
C.	Hirschsprung’s disease
D.	Pelvic trauma
E.	Diverticular disease
F.	Chronic laxative abuse
G.	Colorectal carcinoma
H.	Hypercalcaemia
I.	Parkinson’s disease
J.	Adverse effect of drugs
A

G. Colorectal carcinoma

This a rectal carcinoma. Tenesmus, blood and mucus PR alongside weight loss and anorexia are all highly suggestive. Treatment of rectal carcinoma involves surgical excision where possible. This can either be an anterior resection (tumours in the upper 1/3 of the rectum) or an abdominoperineal resection (if the tumour lies lower down). APER involves the formation of a permanent colostomy and has a high incidence of sexual and urinary dysfunction. Anterior resection involves a colo-anal anastamosis.

49
Q

A 70 year old woman has seen her GP for depression on several occasions. She now complains of abdominal pain, constipation & thirst

For each of the following patients choose the most likely diagnosis from the list of options.
A.	Irritable bowel syndrome
B.	Hypothyroidism
C.	Hirschsprung’s disease
D.	Pelvic trauma
E.	Diverticular disease
F.	Chronic laxative abuse
G.	Colorectal carcinoma
H.	Hypercalcaemia
I.	Parkinson’s disease
J.	Adverse effect of drugs
A

H. Hypercalcaemia

Symptoms of high calcium include confusion, constipation, polyuria, polydipsia, depression, kidney stones and lethargy. This can be remembered by ‘stones, bones, abdominal groans and psychiatric moans’. 90% of hypercalcaemia is caused by primary hyperparathyroidism or cancer. Malignancy can cause hypercalcaemia either by direct bony involvement leading to osteolytic lesions or paraneoplastic syndromes involving PTHrp release. The tumour is typically very advanced if hypercalcaemia is a feature. Less common causes include vitamin D overdose, hyperthyroidism, immobilisation, Paget’s and milk-alkali syndrome. The classic bone disease in hyperparathyroidism is osteitis fibrosa cystica which causes pain. The serum PTH level is elevated in primary hyperparathyroidism whereas it may be very low in malignancy due to negative feedback.

50
Q

A 30 year old male intravenous drug user with a history of tuberculosis develops profuse watery diarrhoea with no abdominal pain.

For each of the following patients choose the most likely diagnosis from the list of options.
A. Ulcerative colitis
B. Crohn’s disease
C. Chronic pancreatitis
D. Staphylococcus aureus
E. Cryptosporidium infection
F. Campylobacter
G. Villous adenoma of the rectum
H. Irritable bowel syndrome
I. Diverticular disease
J. Drug-induced diarrhoea
K. Clostridium difficile
L. Coeliac disease
A

E. Cryptosporidium infection

I’m sure at some point most of you will see ‘diarrhoea’ and ‘HIV’ together and jump straight to Cryptosporidium. This is a disease caused by a protozoa and diagnosis is made in the lab by detection of oocysts or antigens in stool. The presentation is of watery diarrhoea, often accompanied with severe pain in the tummy, often lasting more than 7 days. It is self-limiting if the patient is immunocompetent but those who are immunocompromised can suffer a chronic sveere course. Those most at risk are those with T cell deficiencies, such as HIV, and those with haematological malignancies, especially children. In immunocompetent people, nitazoxanide can be used in treatment (as can paramomycin). In immunocompromised patients, treatment mainly aims at treating the primary disorder, such as using HAART to improve CD4 cell count and to restore immunity. Protease inhibitors such as ritanovir also act directly to reduce host cell invasion by the sporozoites and reduce parasite development.

51
Q

A 25 year old male student presents with 12 hours of abdominal pain, vomiting & watery diarrhoea. This has occurred once before.

For each of the following patients choose the most likely diagnosis from the list of options.
A. Ulcerative colitis
B. Crohn’s disease
C. Chronic pancreatitis
D. Staphylococcus aureus
E. Cryptosporidium infection
F. Campylobacter
G. Villous adenoma of the rectum
H. Irritable bowel syndrome
I. Diverticular disease
J. Drug-induced diarrhoea
K. Clostridium difficile
L. Coeliac disease
A

F. Campylobacter

This patient has infectious vomiting predominant food poisoning. When vomiting is the main presenting symptom, you should be thinking of Staphylococcus aureus, Bacillus cereus or norovirus. There is a short history in a previous well person. This man has probably eaten something dodgy like a kebab with undercooked chicken, or something like that. The mainstay of treatment is rehydration and supportive therapy. Antibiotics may be indicated, particularly in severe cases.

52
Q

A 27 year old woman presents with a 2 week history of bloody diarrhoea & abdominal pain. She has also passed mucus per rectum at times

For each of the following patients choose the most likely diagnosis from the list of options.
A. Ulcerative colitis
B. Crohn’s disease
C. Chronic pancreatitis
D. Staphylococcus aureus
E. Cryptosporidium infection
F. Campylobacter
G. Villous adenoma of the rectum
H. Irritable bowel syndrome
I. Diverticular disease
J. Drug-induced diarrhoea
K. Clostridium difficile
L. Coeliac disease
A

A. Ulcerative colitis

While this could be Crohn’s disease, bloody diarrhoea is more commonly a presentation of UC than Crohn’s. UC is characterised by diffuse mucosal inflammation running a relapsing and remitting course. Bloody diarrhoea is commonly experienced by patients who may also complain of other symptoms such as (lower) abdominal pain, faecal urgency and the host of extra-intestinal manifestations associated with UC. Diagnosis of UC requires endoscopy with biopsy and a negative stool culture to rule out infectious gastroenteritis. Flare ups are usually linked to pathogens so a stool culture will always be needed in these cases. Toxic megacolon is a complication which is associated with a risk of perforation. UC is also linked with bowel adenocarinoma and PSC. Treatment involves mesalazine (5-ASA) used to induce and maintain remission.

53
Q

A 45 year old man has a long history of drinking excess alcohol. He has a 3 month history of intermittent, severe abdominal pain & diarrhoea with pale, bulky, foul-smelling stools which are hard to flush away

For each of the following patients choose the most likely diagnosis from the list of options.
A. Ulcerative colitis
B. Crohn’s disease
C. Chronic pancreatitis
D. Staphylococcus aureus
E. Cryptosporidium infection
F. Campylobacter
G. Villous adenoma of the rectum
H. Irritable bowel syndrome
I. Diverticular disease
J. Drug-induced diarrhoea
K. Clostridium difficile
L. Coeliac disease
A

C. Chronic pancreatitis

This is chronic pancreatitis which is most commonly associated with chronic alcohol abuse. Features include the epigastric pain here, which classically radiates to the back, and steatorrhoea from malabsorption, described here with the pale, foul-smelling and difficult to flush stools. There may additionally be DM due to pancreatic failure and the patient may be malnourished. The diagnosis is based on findings and imaging – your options are USS which is less sensitive, or CT, which is more sensitive but involves radiation exposure. AXR is not a sensitive enough test. There is no real definitive treatment, which is mainly symptomatic and the underlying and precipitating factors are treated – in this case, this man’s alcohol excess. Complications of chronic pancreatic imflammation include the development of pseudocysts, calficiation, DM and malabsorption.

54
Q

A 55 year old man who takes bendrofluazide for hypertension, presents with a 2 month history of watery diarrhoea with occasional blood & mucus mixed in the stool. He has serum potassium of 2.3mmol/l.

For each of the following patients choose the most likely diagnosis from the list of options.
A. Ulcerative colitis
B. Crohn’s disease
C. Chronic pancreatitis
D. Staphylococcus aureus
E. Cryptosporidium infection
F. Campylobacter
G. Villous adenoma of the rectum
H. Irritable bowel syndrome
I. Diverticular disease
J. Drug-induced diarrhoea
K. Clostridium difficile
L. Coeliac disease
A

G. Villous adenoma of the rectum

Do not be fooled by the bendrofluazide, which is taken by many patients, but does not by itself lead to such profound hypokalaemia (but I’m sure contributes), and in any case would not account for the GI symptoms experienced. This patient has a villous adenoma, which is a type of polyp in the GIT with a malignant potential. Most colorectal cancers arise from an adenoma and polypectomy reduces the incidence of colorectal cancer. The non-neoplastic polyps include hyperplastic ones, hamartomas, inflammatory and lymphoid polyps. Villous adenomas secrete large amounts of mucus and result in hypokalaemia.

Adenomatous polyps are increasingly common with age. FAP and Gardner’s syndrome, for example, predispose to adenomatous polyps (Peutz-Jeghers leads to hamartomatous polyps). This patient is going to need a colonoscopy with biopsy histology to see if the polyp is benign or malignant.

55
Q

A 40 year old man has just returned from a holiday in Kenya. Since his return, he has developed watery diarrhoea with crampy abdominal pain.

For each of the following patients choose the most likely diagnosis from the list of options.
A. Ulcerative colitis
B. Crohn’s disease
C. Chronic pancreatitis
D. Staphylococcus aureus
E. Cryptosporidium infection
F. Campylobacter
G. Villous adenoma of the rectum
H. Irritable bowel syndrome
I. Diverticular disease
J. Drug-induced diarrhoea
K. Clostridium difficile
L. Coeliac disease
A

F. Campylobacter

So why is this Campylobacter? Well, it doesn’t have to be. Salmonella, E. Coli, Shigella, Listeria, Vibrio species etc all present with symptoms which are not drastically different and the only way to be sure is to do a stool culture. The only real option here are between Campylobacter, Staphylococcus aureus and Clostridium difficile. However, this patient does not have a history of recent antibiotic use. Staphylococcus tends to present with vomiting as the main feature and the watery diarrhoea here is typical of Campylobacter. UC and CD are chronic conditions (it is worth noting that Yersinia enterocolitis can mimic Crohn’s RLQ pain). This person has most likely eaten something dodgy on holiday. Erythromycin can be used effectively if started early but resistance is a problem and only a small number will benefit. Campylobacter jejuni is the main cause of food poisoning (also coli and fetus species cause disease). Diarrhoea normally resolves in 5-7 days and the patient will need fluid/electrolyte replacement. Campylbacter is one of the infections which is commonly linked to Guillain-Barre (although still a rare phenomenon).

56
Q

A 21 year old student has been on a drinking binge to celebrate the end of his final exams. He has a 6 hour history of profuse vomiting with small amounts of fresh blood mixed in the vomit. His vital signs are stable.

For each of the following patients choose the most likely diagnosis from the list of options.
A.	Angiodysplasia
B.	Gastric erosions
C.	Peptic ulcer
D.	Mallory-Weiss syndrome
E.	Osler-Weber-Rendu syndrome
F.	Oesophageal carcinoma
G.	Mallory-Weiss syndrome
H.	GORD
I.	Peutz-Jeghers syndrome
J.	Bleeding diathesis
K.	Carcinoma of the stomach
L.	Oesophageal varices
A

D. Mallory-Weiss syndrome

This occurs after a rise in abdominal pressure which induces a tear in the oesophageal mucosa, causing subsequent GI bleeding. It commonly presents with haematemesis after an episode of retching/vomiting/coughing/straining. Hence, risk factors include anything which can cause vomiting like heavy alcohol use, which is commonly the case in EMQs. Also, other conditions would include food poisoning, bowel obstruction, hyperemesis gravidarum, bulimia, the chronic cough of COPD, meningitis etc… you name it really. Classically, MWT presents with a small self limiting episode of haematemesis. Definitive diagnosis is made by OGD. Treatment is supportive because most cases, as mentioned, are self limiting and emergency treatment is not offered unless the patient is showing signs of clinical instability. If the patient is actively bleeding, treatment will be with therapeutic endoscopy in most cases, and very very few cases will require more intervention such as angiography with embolisation.

57
Q

A 24 year old woman has had 24 hours of vomiting & diarrhoea, which she thinks followed eating reheated take-away food. There was fresh blood in the last 3 vomits. Vital signs are stable.

For each of the following patients choose the most likely diagnosis from the list of options.
A.	Angiodysplasia
B.	Gastric erosions
C.	Peptic ulcer
D.	Mallory-Weiss syndrome
E.	Osler-Weber-Rendu syndrome
F.	Oesophageal carcinoma
G.	Mallory-Weiss syndrome
H.	GORD
I.	Peutz-Jeghers syndrome
J.	Bleeding diathesis
K.	Carcinoma of the stomach
L.	Oesophageal varices
A

D. Mallory-Weiss syndrome

This occurs after a rise in abdominal pressure which induces a tear in the oesophageal mucosa, causing subsequent GI bleeding. It commonly presents with haematemesis after an episode of retching/vomiting/coughing/straining. Hence, risk factors include anything which can cause vomiting like heavy alcohol use, which is commonly the case in EMQs. Also, other conditions would include food poisoning, bowel obstruction, hyperemesis gravidarum, bulimia, the chronic cough of COPD, meningitis etc… you name it really. Classically, MWT presents with a small self limiting episode of haematemesis. Definitive diagnosis is made by OGD. Treatment is supportive because most cases, as mentioned, are self limiting and emergency treatment is not offered unless the patient is showing signs of clinical instability. If the patient is actively bleeding, treatment will be with therapeutic endoscopy in most cases, and very very few cases will require more intervention such as angiography with embolisation.

58
Q

A 55 year old man has chronic back pain for which he takes diclofenac. He has epigastric pain after meals & has recently developed black tarry stools & has had an episode of ‘coffee-ground’ vomiting.

For each of the following patients choose the most likely diagnosis from the list of options.
A.	Angiodysplasia
B.	Gastric erosions
C.	Peptic ulcer
D.	Mallory-Weiss syndrome
E.	Osler-Weber-Rendu syndrome
F.	Oesophageal carcinoma
G.	Mallory-Weiss syndrome
H.	GORD
I.	Peutz-Jeghers syndrome
J.	Bleeding diathesis
K.	Carcinoma of the stomach
L.	Oesophageal varices
A

C. Peptic ulcer

This patient has a bleeding peptic ulcer (the black tarry stools and the coffee-ground vomit from the UGI bleed). Epigastric pain and tenderness related to eating a meal is typical of a peptic ulcer. 80% are duodenal and 20% are gastric. Ulcers may cause iron deficiency anaemia and associated symptoms may feature. Key risk factors are NSAID use, like in this patient, H. pylori infection, smoking and a family history of PUD. Zollinger-Ellison syndrome should be considered if there are multiple ulcers or ulcers refractory to treatment.

Gastric ulcers classically cause pain which is exacerbated by eating and immediately relieved on vomiting. There is usually also weight loss due to a fear of food and its association with pain. Duodenal ulcers are classically made worse by hunger and are relieved by eating and the patient may wake at night with the pain. As a result, weight gain is typically a feature. In reality, it is difficult to differentiate the site of the ulcer based on these features.

The most specific and sensitive test is an upper GI endoscopy which is initially ordered if the patient has ‘red flag’ symptoms, is >55 years of age or fails to respond to treatment. Duodenal ulcers rarely undergo malignant transformation so do not require a compulsory biopsy but gastric ulcers require biopsies to rule this out. In patients who are 55 or younger without ‘red flags’, testing for Helicobacter pylori (breath testing with radiolabelled urea or stool antigen testing) is necessary. Management is aimed at correcting the underlying cause such as discontinuing NSAIDs. H. pylori eradication should be started if the organism is present with triple therapy. Otherwise, a PPI is indicated.

59
Q

A 35 year old man with a long history of excess drinking of alcohol presents with massive haematemesis. He is also jaundiced, hypotensive & a tachycardia.

For each of the following patients choose the most likely diagnosis from the list of options.
A.	Angiodysplasia
B.	Gastric erosions
C.	Peptic ulcer
D.	Mallory-Weiss syndrome
E.	Osler-Weber-Rendu syndrome
F.	Oesophageal carcinoma
G.	Mallory-Weiss syndrome
H.	GORD
I.	Peutz-Jeghers syndrome
J.	Bleeding diathesis
K.	Carcinoma of the stomach
L.	Oesophageal varices
A

L. Oesophageal varices

Oesophageal varices occurs as a result of portal hypertension which is a complication of cirrhosis, caused in this patient by his long history of alcohol excess. Other signs may be present such as spider naevi, ascites, caput medusa (vascular collaterals in the abdominal wall), jaundice etc. Splenomegaly is also commonly found and hence patents often have thrombocytopenia and anaemia as a result. The bleeding carries a significant morbidity and mortality, and beta-blockers and/or endoscopic ligation can prevent variceal bleeding prophylactically (though beta blockers are not be used in the acute setting of a variceal bleed – do not get confused here!). Oesophageal varices are basically dilated veins and these can be seen on OGD. Worldwide, HBV and HCV are also major causes of cirrhosis, leading to varices and HIV co-infection can rapidly speed up the progression to cirrhosis in chronic liver failure. The size of the varices is the key predictor of haemorrhage. Acute bleed can be managed with resuscitation, terlipression (DDAVP)/somatostatin analogues/endoscopic ligation. Additionally, a shunt can be deployed and antbiotic prophylaxis started.

60
Q

52 year old man presents with abdominal distension & ankle swelling. He has been drinking 6 pints of beer & half a bottle of whisky a day for some years. On examination he has palmar erythema & spider naevi on his chest.

For each patient below, choose the most likely diagnosis from the list of options.
A.	Budd Chiari syndrome
B.	Primary biliary cirrhosis
C.	Liver cirrhosis
D.	Secondary liver tumours
E.	Carcinoma of caecum with peritoneal secondaries
F.	Nephrotic syndrome
G.	Heart failure
H.	Primary liver tumour
I.	Tuberculous peritonitis
J.	Carcinoma of the ovary
K.	Bacterial peritonitis
A

C. Liver cirrhosis

Cirrhosis is the end-stage of chronic liver disease, in this case due to alcoholic liver disease. Cirrhosis results in hepatic insufficiency and portal hypertension. This has resulted in the patient’s ascites which is a symptom of decompensated cirrhosis. Other complications include variceal bleeds, jaundice, hepatic encephalopathy, hepatorenal syndrome and the development of HCC. Palmar erythema affects the thenar and hypothenar eminences. Apart from spider naevi and palmar erythema, other signs you might find include telangiectasia, bruising, gynaecomastia, Dupuytren’s contracture, parotid swelling and a red tongue. The patient will have to undergo a diagnostic paracentesis for the ascitic presentation. The treatment of ascites involves restricting salt intake and the use of diuretics (frusemide and spironolactone).

61
Q

A 71 year old man who has had a MI 6 months ago presents with shortness of breath & fatigue. On examination, the JVP is raised. He has pitting oedema to the knees. There is tenderness in the right upper quadrant with a smooth liver edge at 5cm.

For each patient below, choose the most likely diagnosis from the list of options.
A.	Budd Chiari syndrome
B.	Primary biliary cirrhosis
C.	Liver cirrhosis
D.	Secondary liver tumours
E.	Carcinoma of caecum with peritoneal secondaries
F.	Nephrotic syndrome
G.	Heart failure
H.	Primary liver tumour
I.	Tuberculous peritonitis
J.	Carcinoma of the ovary
K.	Bacterial peritonitis
A

G. Heart failure

This patient has heart failure, which has possibly occured as a consequence of his MI. SOB indicates pulmonary oedema due to LV failure. The raised JVP, peripheral oedema and tender hepatomegaly indicates RV failure. Initial investigations should include ECG, CXR, TTE and bloods including BNP levels. First line treatment is with an ACE inhibitor which reduces morbidity and mortality. Salt and fluid restriction is also beneficial. All patients with chronic heart failure will also receive a beta blocker such as carvedilol. Other adjuncts include spironolactone, diuretics, hydralazine and a nitrate, and digoxin.

62
Q

A 50 year old man with a 1 month history of progressive abdominal distension preceded by increased tiredness, shortness of breath on exertion & weight loss of 10kg. There is a non-tender irregular mass in the right iliac fossa.

For each patient below, choose the most likely diagnosis from the list of options.
A.	Budd Chiari syndrome
B.	Primary biliary cirrhosis
C.	Liver cirrhosis
D.	Secondary liver tumours
E.	Carcinoma of caecum with peritoneal secondaries
F.	Nephrotic syndrome
G.	Heart failure
H.	Primary liver tumour
I.	Tuberculous peritonitis
J.	Carcinoma of the ovary
K.	Bacterial peritonitis
A

E. Carcinoma of caecum with peritoneal secondaries

The weight loss of 10kg, fatigue and non-tender irregular RIF mass point to caecal carcinoma. Right sided colorectal cancer tends to present with anaemic symptoms. Almost 90% are anaemic at diagnosis. The progressive abdominal distension indicates the presence of peritoneal secondaries, which causes vague symptoms. Treatment in this case for a cancer that has become widely metastatic will be palliative.

63
Q

A 20 year old man presents with generalised swelling of the limbs, face & abdomen. Tests show that he has a normal blood count & liver function tests, except a low albumin level of 18g/L and a high cholesterol of 9.5mmol/L.

For each patient below, choose the most likely diagnosis from the list of options.
A.	Budd Chiari syndrome
B.	Primary biliary cirrhosis
C.	Liver cirrhosis
D.	Secondary liver tumours
E.	Carcinoma of caecum with peritoneal secondaries
F.	Nephrotic syndrome
G.	Heart failure
H.	Primary liver tumour
I.	Tuberculous peritonitis
J.	Carcinoma of the ovary
K.	Bacterial peritonitis
A

F. Nephrotic syndrome

Nephrotic syndrome is the triad of proteinuria (>3.5g/24h), hypoalbuminaemia (<30g/L) and oedema. Some definitions also include hyperlipidaemia which is a feature. It is worth noting that this in in contrast to nephritic syndrome, which features acute kidney injury and red cells and red cell casts in urine. A patient who presents with just proteinuria is described as having ‘nephrotic-range proteinuria’. The most common cause in children is minimal change GN, in younger adults it is FSGS and the most common cause in older people is membranous nephropathy. The most common cause in those with a history of long standing DM is diabetic nephropathy.

A disease process disrupts the glomerular filtration barrier leading to proteinuria. Urinary loss of albumin leads to hypoalbuminaemia. The liver goes into overdrive to try to compensate for this protein loss by increasing synthesis of albumin as well as LDL, vLDL and lipoproteins. This causes the dyslipidaemia. The oedema results from the lowered oncotic pressure which results from the low albumin levels. There is also an increased risk of infection in these individuals due to the loss of complement and Ig in the urine.

64
Q

A 50 year old alcoholic man fails to respond to treatment for pancreatitis and has recurrent epigastric pain. There is a palpable epigastric mass. CT scan of the abdomen shows a round well-circumscribed mass in the epigastrium.

From each patient below, choose the most likely diagnosis from the list of options.
A.	Divarication of the recti
B.	Pancreatic pseudocyst
C.	Perforated peptic ulcer
D.	Pancreatic abscess
E.	Fractured rib
F.	Oesophageal varices
G.	Hepatoma
H.	Mallory-Weiss tear
I.	Haematoma of the rectus sheath
J.	Sigmoid volvulus
K.	Umbilical hernia
L.	Pancreatic ascites
M.	Pancratic effusion
N.	Splenic rupture
A

B. Pancreatic pseudocyst

This patient has developed a pancreatic psuedocyst as a complication of pancreatitis. Pseudocysts are collections of fluids with a high concentration of enzymes. The walls are fibrotic membranes of the peritoneum, mesentery and serosa which stops the fluid from leaking out. The wall is not epithelium and indeed there is no epithelial lining – it is not a real cyst. In patients who fail to respond to treatment, this should be considered as a possible diagnosis. The most common finding is pain, followed by a palpable mass. CT scan is diagnostic. Treatment options include excision, drainage (surgical or percutaneous, or internal e.g. cystojejunostomy Roux-en-Y etc, which I’m sure is going into too much surgery than is necessary now but surgical wannabes can look up the procedures… if they really want to – also cystogastrostomy and cystoduodenostomy).

The pseudocyst can be complicated with infection, rupture and haemorrhage. Pancreatic abscess would give a fever and CT will show a ring-enhancing fluid collection with gas. Treatment would be drainage and antibiotics. Pancreatic ascites is pancreatic fluid accumulating in the peritoneal cavity due to chronic pseudocyst leakage (in most cases, anyway) – there will be weight loss and the ascites will not respond to diuretics. A pancreatic effusion is secondary to a fistula draining into the chest, from the pancreas. These are actually all more common complications to arise from pancreatitis than a pseudocyst.

65
Q

A 40 year old multiparous woman presents with a midline abdominal mass. The mass is non tender & appears when she is straining. On examination, the midline mass is visible when she raises her head off the examining bed.

From each patient below, choose the most likely diagnosis from the list of options.
A.	Divarication of the recti
B.	Pancreatic pseudocyst
C.	Perforated peptic ulcer
D.	Pancreatic abscess
E.	Fractured rib
F.	Oesophageal varices
G.	Hepatoma
H.	Mallory-Weiss tear
I.	Haematoma of the rectus sheath
J.	Sigmoid volvulus
K.	Umbilical hernia
L.	Pancreatic ascites
M.	Pancratic effusion
N.	Splenic rupture
A

A. Divarication of the recti

Divaricate means to spread apart. The rectus goes from the pubic crest, tubercle and symphysis to the costal cartilages 5,6 and 7, costal margin of 7, sternum and diaphragm. It is innervated by T7-12. Normally, the rectus muscles meet in the midline (linea alba). However, some people have a defect above the umbilicus which causes the gap between the recti to be wider than normal. Hence, when the patient sites up, the rectus muscles will spread apart. Surgica correction is possible but most are asymptomatic. It is not a true hernia, and this is the only option from the list that fits the presentation.

66
Q

A 19 year old man presents with sudden severe upper abdominal pain after being tackled during rugby practice. He was recently diagnosed with glandular fever.

From each patient below, choose the most likely diagnosis from the list of options.
A.	Divarication of the recti
B.	Pancreatic pseudocyst
C.	Perforated peptic ulcer
D.	Pancreatic abscess
E.	Fractured rib
F.	Oesophageal varices
G.	Hepatoma
H.	Mallory-Weiss tear
I.	Haematoma of the rectus sheath
J.	Sigmoid volvulus
K.	Umbilical hernia
L.	Pancreatic ascites
M.	Pancratic effusion
N.	Splenic rupture
A

N. Splenic rupture

Infectious mononucleosis caused by EBV infection causes splenomegaly. This makes the patient susceptible to splenic rupture due to trauma, such as during this rugby practice session. Rupture is a cause of splenomegaly, and splenomegaly is a risk factor for rupture. If the patient is previously known to have mono, with an enlarged spleen, then they should really avoid contact sports. It would be an irresponsible doctor to not advise against this. This is a medical emergency as the spleen is a very vascular organ and bleeding can rapidly lead to shock and death.

67
Q

An 87 year old woman presents with constipation and nausea.

From each patient below, choose the most likely diagnosis from the list of options.
A.	Divarication of the recti
B.	Pancreatic pseudocyst
C.	Perforated peptic ulcer
D.	Pancreatic abscess
E.	Fractured rib
F.	Oesophageal varices
G.	Hepatoma
H.	Mallory-Weiss tear
I.	Haematoma of the rectus sheath
J.	Sigmoid volvulus
K.	Umbilical hernia
L.	Pancreatic ascites
M.	Pancratic effusion
N.	Splenic rupture
A

J. Sigmoid volvulus

While this woman has indeed presented with very non-specific symptoms, the only feasible answer from the list is a sigmoid volvulus. A volvulus is bowel obstruction occuring due to a loop of bowel twisting on its own mesenteric axis. Broadly speaking, there are three types: small bowel, sigmoid and gastric. This is something you need to be able to recognise on AXR and it appears as a dilated loop of large bowel present in the lower abdomen, resembling a coffee bean shape (or like an upside down U shape). The rest of the bowel is usually dilated. For a caecal volvulus, the caecum leaves the RLQ to appear like a second satomach bubble in the centre of the film. There is often associated small bowel dilation. A gastric volvulus is very rare.

68
Q

A 55 year old male alcoholic presents with vomiting 800ml of blood. His blood pressure is 80/50 with a pulse rate of 120. He also has ascites.

From each patient below, choose the most likely diagnosis from the list of options.
A.	Divarication of the recti
B.	Pancreatic pseudocyst
C.	Perforated peptic ulcer
D.	Pancreatic abscess
E.	Fractured rib
F.	Oesophageal varices
G.	Hepatoma
H.	Mallory-Weiss tear
I.	Haematoma of the rectus sheath
J.	Sigmoid volvulus
K.	Umbilical hernia
L.	Pancreatic ascites
M.	Pancratic effusion
N.	Splenic rupture
A

F. Oesophageal varices

Oesophageal varices occurs as a result of portal hypertension which is a complication of cirrhosis, caused in this patient by his long history of alcohol excess. Other signs may be present such as spider naevi, ascites, caput medusa (vascular collaterals in the abdominal wall), jaundice etc. Splenomegaly is also commonly found and hence patents often have thrombocytopenia and anaemia as a result. The bleeding (explaining the BP and HR of this patient) carries a significant morbidity and mortality, and beta-blockers and/or endoscopic ligation can prevent variceal bleeding prophylactically (though beta blockers are not be used in the acute setting of a variceal bleed – do not get confused here!). Oesophageal varices are basically dilated veins and these can be seen on OGD. Worldwide, HBV and HCV are also major causes of cirrhosis, leading to varices and HIV co-infection can rapidly speed up the progression to cirrhosis in chronic liver failure. The size of the varices is the key predictor of haemorrhage. Acute bleed can be managed with resuscitation, terlipression (DDAVP)/somatostatin analogues/endoscopic ligation. Additionally, a shunt can be deployed and antbiotic prophylaxis started.

69
Q

34 year old male with Crohn’s disease had had no bowel motions for 4 days & has been vomiting for 24 hours. Examination reveals a distended abdomen & tinkling bowel sounds.

For each patient below, choose the most likely diagnosis from the list.
A.	Oesophageal carcinoma
B.	Combined oral contraceptive pill
C.	Salmonella
D.	Appendicitis
E.	Bowel obstruction
F.	Pyloric stenosis
G.	Peptic ulcer disease
H.	Viral gastroenteritis
I.	Pancreatitis
J.	Gastric carcinoma
K.	Intussusception
L.	Uraemia
M.	Bulimia
A

E. Bowel obstruction

Bowel obstruction has several causes such as adhesions or cancer. The lack of bowel motions, vomiting, distended abdomen and tinkling bowel signs are all indicative. The proximal segment of bowel dilates and distal bowel collapses. Completely obstructed patients generally require surgery. If, on AXR, air is seen to be seeping past the obstruction then the obstruction is partial. As a standard, all patients should be made NBM and given supplemental oxygen, IV fluids and NG decompression (to reduce flow/gastric contents/air towards the obstruction), unless they are rushed off for an emergency laparotomy because, for example, they have complete SBO and are peritonitic.

70
Q

83 years old man with longstanding heart failure for which he takes digoxin & diuretics. For the last 24 hours he has been vomiting & passed very little urine. On examination he is pale & mildly dehydrated; examination of the abdomen is normal.

For each patient below, choose the most likely diagnosis from the list.
A.	Oesophageal carcinoma
B.	Combined oral contraceptive pill
C.	Salmonella
D.	Appendicitis
E.	Bowel obstruction
F.	Pyloric stenosis
G.	Peptic ulcer disease
H.	Viral gastroenteritis
I.	Pancreatitis
J.	Gastric carcinoma
K.	Intussusception
L.	Uraemia
M.	Bulimia
A

L. Uraemia

This patient has developed acute renal failure, probably associated with the longstanding CCF. Advanced heart failure will lead to depressed renal perfusion and ARF. The decreased urine output is a symptom and the vomiting here is caused by uraemia or a general build up of waste products. An acute increase in creatinine will be seen, commonly with hyperkalaemia, hyperphosphataemia and a metabolic acidosis. There may also be respiratory compensation for this. Treatment is largely supportive, managing, in this case, the heart failure, and correcting abnormalities like volume status and the metabolic acidosis. Dialysis may be required.

71
Q

54 year old publican has 48 hour history of severe epigastric pain & vomiting. On examination he is unwell. Pulse rate is 110/min, BP 130/90. Temp 38 degrees Celsius. Upper abdomen very tender. Amylase 1000U/l.

For each patient below, choose the most likely diagnosis from the list.
A.	Oesophageal carcinoma
B.	Combined oral contraceptive pill
C.	Salmonella
D.	Appendicitis
E.	Bowel obstruction
F.	Pyloric stenosis
G.	Peptic ulcer disease
H.	Viral gastroenteritis
I.	Pancreatitis
J.	Gastric carcinoma
K.	Intussusception
L.	Uraemia
M.	Bulimia
A

I. Pancreatitis

This patient has acute pancreatitis. He has vomited and is describing mid-epigastric pain radiating around to the back. This patient can also have nausea and vomiting too, with agitation and confusion. A pleural effusion is seen in half of patients with acute pancreatitis. Complicated haemorrhagic pancreatitis may exhibit Cullen’s sign, Grey-Turner’s sign and Fox’s sign. Make sure you know what these are and you are familiar with the causes of acute pancreatitis (GET SMASHED). Those caused by hypocalcaemia may display Chvostek’s sign and Trousseau’s sign.

Key to diagnosis is serum amylase or lipase levels which are massively elevated. Prognostic criteria are outlined in Ranson’s criteria applied on admission and after 48 hours, or the modified Glasgow score which you can find in your Oxford Handbook. An abdominal CT is however the most sensitive and specific study and findings may include enlargement of the pancreas with irregular contours, necrosis, pseudocysts and peripancreatic fat obliteration.

For interest, urinary trypsinogen-2 is now considered a better screening test than amylase but is not currently clinically used.

72
Q

34 year old man has had vomiting 2-3 times a day for 3 days. Complains of severe crampy abdominal pain & blood stained watery diarrhoea. On examination temp 37.7 degrees Celsius, abdomen soft; complains of generalised tenderness. No masses/rebound/guarding.

For each patient below, choose the most likely diagnosis from the list.
A.	Oesophageal carcinoma
B.	Combined oral contraceptive pill
C.	Salmonella
D.	Appendicitis
E.	Bowel obstruction
F.	Pyloric stenosis
G.	Peptic ulcer disease
H.	Viral gastroenteritis
I.	Pancreatitis
J.	Gastric carcinoma
K.	Intussusception
L.	Uraemia
M.	Bulimia
A

C. Salmonella

The only option on the list which fits is Salmonella. This could obviously be E coli or Campylobacter too, for instance. This is infectious gastroenteritis and Salmonella is a common cause, pretty much able to contaminate any food. It is commonly linked to poultry, dairy items and undercooked eggs. It is self-limiting and diagnosis is on isolating the organism from a stool culture. Treatment is supportive with fluid and electrolyte replacement and antibiotics are generally used only for patients with risk factors for severe disease or those with extra-GI complications.

73
Q

A 25 year old travelling salesman is awoken in his hotel with crampy abdominal pain, feeling very ill & vomits 3 times over the next half an hour. He asks the receptionist to call a doctor.

For each patient below, choose the most likely diagnosis from the list.
A.	Oesophageal carcinoma
B.	Combined oral contraceptive pill
C.	Salmonella
D.	Appendicitis
E.	Bowel obstruction
F.	Pyloric stenosis
G.	Peptic ulcer disease
H.	Viral gastroenteritis
I.	Pancreatitis
J.	Gastric carcinoma
K.	Intussusception
L.	Uraemia
M.	Bulimia
A

H. Viral gastroenteritis

Viral gastroenteritis often presents with mainly UGI symptoms like N&V more so than diarrhoea. It is on this basis that this is likely to be viral gastroenteritis caused by organisms such as rotavirus, norovirus and adenovirus. Staphylococcus aureus and Bacillus cereus also tend to cause mostly UGI symptoms and acts by preformed toxins, though these are not on the list of available options. The differentiation is made on studying the stool and identifying the organism.

74
Q

A 30 year old woman who has recently returned from holiday in the Gambia. She is in the 3rd trimester of pregnancy & complains of headaches & fever. On examination her BP is 110/70, there is a soft ESM, shotty lymphadenopathy & hepatosplenomegaly. Examination of the skin is unremarkable. Investigations revealed a Hb of 10.5g/dl, WBC of 5x109/l, platelet count of 80x109/l.

For each of the patients below, choose the most likely diagnosis from the list of options.
A.	RA
B.	Severe emphysema
C.	CLL
D.	Polycythaemia rubra vera
E.	Toxoplasmosis
F.	CML
G.	Haemachromatosis
H.	Portal vein thrombosis
I.	Congestive cardiac failure
J.	Right heart failure
K.	Cirrhosis with hepatoma
L.	Malignant melanoma
M.	Malaria
N.	Systemic amyloidosis
A

M. Malaria

In the Western world, almost all cases of malaria occurs in travellers so an adequate travel history is crucial or the diagnosis may be missed. Patients typically present with non-specific symptoms such as a fever, sweats, chills and myalgia. This woman has just returned from an endemic area. Sometimes EMQs will describe patterns of fevers occuring at regular intervals of 48-72 hours associated with P. vivax, P. ovale and P. malariae infections but in most patients there is no specific pattern. Hepatosplenomegaly is a common presenting sign although not common at presentation in a first world setting. Thrombocytopenia is common with falciparum infection and a mild degree of anaemia are commonly seen. WCC can be high, low or normal.

Pregnant women affected by P. falciparum are susceptible to the complications of pregnancy due to placental parasite sequestration. Treatment of malaria in pregnancy must be managed with an ID specialist and should be treated with IV antimalarial therapy.

The test of choice is Giesma-stained thick and thin blood smears. Thick films sensitively detect parasites whereas thin films allow species identification and calculation of parasitaemia to guide treatment. Studies have shown that for P falciparum, the most effective treatment is artesunate which is more effective than quinine without the risk of cinchonism. Numerous studies such as the AQUAMAT study in The Lancet showing that quinine should no longer be the established treatment of choice.

75
Q

A 78 year old woman attends complaining of widespread itching. Examination reveals hepatosplenomegaly. The patient appears plethoric with no lymphadenopathy.

For each of the patients below, choose the most likely diagnosis from the list of options.
A.	RA
B.	Severe emphysema
C.	CLL
D.	Polycythaemia rubra vera
E.	Toxoplasmosis
F.	CML
G.	Haemachromatosis
H.	Portal vein thrombosis
I.	Congestive cardiac failure
J.	Right heart failure
K.	Cirrhosis with hepatoma
L.	Malignant melanoma
M.	Malaria
N.	Systemic amyloidosis
A

D. Polycythaemia rubra vera

PRV is a disease of middle and older age and is strongly associated with the JAK2V617 mutation. Pruritis is a common feature and is often severe and evoked by contact with water. Facial redness and fullness is commonly observed and splenomegaly is a common finding. It is a myeloproliferative disorder with raised Hct, Hb and RBC count. Blood hence becomes very viscous. There is a clear link between Budd-Chiari syndrome and subsequent PRV. Treatment is with venesection. Around 30% will go on to develop myelofibrosis.

76
Q

A 78 year old woman attends complaining of recent onset of tiredness. She is pale, has hepatosplenomegaly and generalised painless lymphadenopathy in the neck, axillae and groin. Coombs’ (DAT) test is positive.

For each of the patients below, choose the most likely diagnosis from the list of options.
A.	RA
B.	Severe emphysema
C.	CLL
D.	Polycythaemia rubra vera
E.	Toxoplasmosis
F.	CML
G.	Haemachromatosis
H.	Portal vein thrombosis
I.	Congestive cardiac failure
J.	Right heart failure
K.	Cirrhosis with hepatoma
L.	Malignant melanoma
M.	Malaria
N.	Systemic amyloidosis
A

C. CLL

This elderly woman has CLL. CLL presents in older adults (generally >60) and is often asymptomatic. Smear cells can be seen in peripheral blood smear and it is associated with a warm type AIHA accounting for her pallor and fatigue (hence the Coombs’ test is positive). Painless lymphadenopathy may be present and splenomegaly is a common finding. A WCC with differential is required to make a diagnosis. An absolute lymphocytosis will be seen. CML is not associated with an AIHA and tends to present at a younger age.

77
Q

A 60 year old woman is found to have hepatomegaly. She has a history of moderate alcohol use. She had an anterior MI 2 years previously. Examination reveals significant ankle oedema, elevated JVP & 1-2 spider naevi on her chest.

For each of the patients below, choose the most likely diagnosis from the list of options.
A.	RA
B.	Severe emphysema
C.	CLL
D.	Polycythaemia rubra vera
E.	Toxoplasmosis
F.	CML
G.	Haemachromatosis
H.	Portal vein thrombosis
I.	Congestive cardiac failure
J.	Right heart failure
K.	Cirrhosis with hepatoma
L.	Malignant melanoma
M.	Malaria
N.	Systemic amyloidosis
A

I. Congestive ccardiac failure

The significant ankle oedema and raised JVP point towards right sided heart failure.The presence of 1-2 spider naevi is completely normal. Initial investigations should include ECG, CXR, TTE and bloods including BNP levels. First line treatment is with an ACE inhibitor which reduces morbidity and mortality. Salt and fluid restriction is also beneficial. All patients with chronic heart failure will also receive a beta blocker such as carvedilol. Other adjuncts include spironolactone, diuretics, hydralazine and a nitrate, and digoxin.

78
Q

A 50 year old male with haemophilia & hepatitis C presents with weight loss & abdominal discomfort. He is mildly icteric with features of chronic liver disease & a large left lobe of the liver

For each of the patients below, choose the most likely diagnosis from the list of options.
A.	RA
B.	Severe emphysema
C.	CLL
D.	Polycythaemia rubra vera
E.	Toxoplasmosis
F.	CML
G.	Haemachromatosis
H.	Portal vein thrombosis
I.	Congestive cardiac failure
J.	Right heart failure
K.	Cirrhosis with hepatoma
L.	Malignant melanoma
M.	Malaria
N.	Systemic amyloidosis
A

K. Cirrhosis with hepatoma

HCV has caused this patient’s hepatic cirrhosis which has resulted in a hepatoma (HCC). It is likely that his HCV infection has resulted from contaminated blood products due to his haemophilia. This was a key problem before blood donor screening took place. Unlike HBV, HCV infection almost always results in cirrhosis before a hepatoma develops. The length of time the patient has HCV is a good correlate to the development of HCC. HCV is also strongly associated with IVDU which accounts for most infections. Treatment of the hepatoma is guided by staging and prognosis. Treatment includes resection, transplant, percutaneous ablation and chemo-embolisation.

79
Q

A 65 year old heavy smoker. He has been progressively short of breath over a few years. He has a smooth liver edge 2cms below the costal margin.

For each of the patients below, choose the most likely diagnosis from the list of options.
A.	RA
B.	Severe emphysema
C.	CLL
D.	Polycythaemia rubra vera
E.	Toxoplasmosis
F.	CML
G.	Haemachromatosis
H.	Portal vein thrombosis
I.	Congestive cardiac failure
J.	Right heart failure
K.	Cirrhosis with hepatoma
L.	Malignant melanoma
M.	Malaria
N.	Systemic amyloidosis
A

B. Severe emphysema

The liver is palpable in this man because severe emphysema has resulted in hyperexpanded lung fields.

80
Q

A 25 year old female trainee solicitor presents complaining of bloating & excessive flatus. She is passing pellet-like stools associated with abdominal pains. Her symptoms have been intermittent for several years

For each patient below, choose the most likely cause of the symptoms from the list of options.
A.	Colorectal cancer
B.	Benign colonic stricture
C.	Irritable bowel syndrome
D.	Diverticular disease
E.	Radiation proctitis
F.	Hyperthyroidism
G.	Infectious diarrhoea
H.	Ischaemic colitis
I.	Inflammatory bowel disease
A

C. Irritable bowel syndrome

IBS is a chronic condition with abdominal pain associated with bowel dysfunction and is a diagnosis of exclusion. The pain or discomfort may be relieved by defecation. Examination is usually unremarkable and the diagnosis is based on the patient’s history in line with the Rome Criteria. If the patient presents with any worrying symptoms, then these will warrant a more thorough investigation. Treatment depends on the patient’s predominant symptoms. Antispasmodics relieve abdominal pain or discomfort but do not affect bowel habit. Examples include peppermint oil and dicycloverine. Laxatives can also be used such as lactulose. Lifestyle and dietary modifications combined with reassurance remain the 1st line intervention for functional bowel disease. IBS is linked with stressful jobs such as working as a secretary and there is a female/male ratio of 2:1. ‘Pellet-like’ stools is an indicator in EMQs.

81
Q

A 75 year old man with a 6 month history of straining at stool. He also thinks he is not emptying his rectum completely. He is passing blood & mucus per rectum. He has some weight loss & anorexia.

For each patient below, choose the most likely cause of the symptoms from the list of options.
A.	Colorectal cancer
B.	Benign colonic stricture
C.	Irritable bowel syndrome
D.	Diverticular disease
E.	Radiation proctitis
F.	Hyperthyroidism
G.	Infectious diarrhoea
H.	Ischaemic colitis
I.	Inflammatory bowel disease
A

A. Colorectal cancer

This a rectal carcinoma. Tenesmus, blood and mucus PR alongside weight loss and anorexia are all highly suggestive. Treatment of rectal carcinoma involves surgical excision where possible. This can either be an anterior resection (tumours in the upper 1/3 of the rectum) or an abdominoperineal resection (if the tumour lies lower down). APER involves the formation of a permanent colostomy and has a high incidence of sexual and urinary dysfunction. Anterior resection involves a colo-anal anastamosis.

82
Q

A 32 year old female presents with a 4 week history of bloody liquid stool with mucus, 9 times a day. She has anorexia, weight loss & anaemia.

For each patient below, choose the most likely cause of the symptoms from the list of options.
A.	Colorectal cancer
B.	Benign colonic stricture
C.	Irritable bowel syndrome
D.	Diverticular disease
E.	Radiation proctitis
F.	Hyperthyroidism
G.	Infectious diarrhoea
H.	Ischaemic colitis
I.	Inflammatory bowel disease
A

I. Inflammatory bowel disease

This patient gives a history of IBD. The history would be more suggestive of UC where the mainstay of treatment is with 5-ASA. A colonoscopy is required to assess the extent of disease and for a definitive diagnosis. Biopsy in CD will show transmural granulomatous inflammation. CD can affect the whole GIT but favours the TI and proximal colon and is macroscopically characterised by skip lesions. UC on the other hand is characterised by the presence of crypt abscesses, which is pathognomic. CD risk is increased 3-4 fold by smoking whereas smoking seems protective in UC. The mainstay of treatment in CD is with steroids and azathioprine to revent relapses and for those suffering side effects of steroid treatment. TNF-alpha inhibitors also have a role. Surgery in CD is only indicated in a small number of patients who bleed, for bowel perforation and cases of complete obstruction. The aim is to rest distal disease by temporarily diverting faecal flow.

83
Q

A 19 year old male returns from a recent back packing holiday in India. He is passing bloody liquid stools about 15 times a day. He has lassitude, anorexia & a temperature of 37.5 degrees Celsius.

For each patient below, choose the most likely cause of the symptoms from the list of options.
A.	Colorectal cancer
B.	Benign colonic stricture
C.	Irritable bowel syndrome
D.	Diverticular disease
E.	Radiation proctitis
F.	Hyperthyroidism
G.	Infectious diarrhoea
H.	Ischaemic colitis
I.	Inflammatory bowel disease
A

G. infectious diarrhoea

This patient has low grade pyrexia and recent foreign travel making infectious diarrhoea very likely. This man has probably eaten something dodgy while on holiday in India. Think of the organisms which cause bloody diarrhoea such as EHEC. E coli is the most common cause of traveller’s diarrhoea. There may be a contact history due to faecal-oral spread. The mainstay of treatment is rehydration and supportive therapy. Antibiotics may be indicated, particularly in severe ETEC.

84
Q

A 65 year old man had an elective aortic aneurysm repair 5 days ago. He now has abdominal distension & left sided abdominal pain. He is passing a small amount of blood & mucus per rectum.

For each patient below, choose the most likely cause of the symptoms from the list of options.
A.	Colorectal cancer
B.	Benign colonic stricture
C.	Irritable bowel syndrome
D.	Diverticular disease
E.	Radiation proctitis
F.	Hyperthyroidism
G.	Infectious diarrhoea
H.	Ischaemic colitis
I.	Inflammatory bowel disease
A

H. Ischaemic colitis

Ischaemic colitis causes focal or diffuse abdominal pain and often has a more insidious onset than mesenteric ischaemia (over several hours or days). The recent operation in the approximate area has resulted in an incomplete blood supply to that part of the bowel. Mesenteric ischaemia and ischaemic colitis all form part of ‘ischaemic bowel disease’.

85
Q

A 35 year old woman has a 10 year history of low retrosternal dysphagia & painless regurgitation of food in the mouth

For each patient below, choose the most likely diagnosis from the list of options.
A.	Cerebrovascular accident
B.	Myasthenia gravis
C.	Carcinoma of oesophagus
D.	Hiatus hernia
E.	Achalasia
F.	Plummer-Vinson syndrome
G.	Pharyngeal pouch
H.	Gastric volvulus
I.	Carcinoma of bronchus
J.	Pneumonia
K.	Thyroid goitre
A

D. Hiatus hernia

A hiatus hernia is where intraabdominal contents protrude through the oesophageal hiatus of the diaphragm. Risk factors inclyde obesity and high intra-abdominal pressure. The condition may be asymptomatic, or it may present with symptoms (which are non-specific) such as heartburn, dysphagia, pain on swallowing, wheezing, hoarseness and chest pain. A CXR is the first test done and may show an air bubble in the wrong place but barium studies are diagnostic and treatment depends on the symptoms and anatomy of the hernia. Hernias can be sliding or rolling (or mixed, or giant), uncomplicated or complicated by, for instance, obstruction and bleeding. Do you know the difference between a sliding and a rolling hiatal hernia?

86
Q

A 65 year old woman has progressive low retrosternal dysphagia, initially to solids, but now also to liquids – coming on for the last 4 months. There has also been loss of appetite & 3kg weight loss.

For each patient below, choose the most likely diagnosis from the list of options.
A.	Cerebrovascular accident
B.	Myasthenia gravis
C.	Carcinoma of oesophagus
D.	Hiatus hernia
E.	Achalasia
F.	Plummer-Vinson syndrome
G.	Pharyngeal pouch
H.	Gastric volvulus
I.	Carcinoma of bronchus
J.	Pneumonia
K.	Thyroid goitre
A

C. Carcinoma of oesophagus

Dysphagia (in this progressive pattern) coupled with weight loss points to malignancy. Dysphagia occurs when there is obstruction of more than 2/3 of the lumen and presence indicates locally advanced disease. There may additionally be odynophagia. Men are twice as likely to develop oesophageal cancer. GORD, Barrett’s oesophagus, FH, tobacco and alcohol are all risk factors. The two main types are squamous cell carcinoma and adenocarcinoma. Tumours in the upper 2/3 of the oesophagus are SCC whereas those that lie in the lower 1/3 are adenocarcinomas. The main test to order is an OGD with biopsy. Treatment is either surgical resection or with chemo or radiotherapy alongside endoscopic ablation with or without stenting and brachytherapy.

87
Q

A 45 year old lady presents with high retrosternal dysphagia. She has spoon-shaped nails & is noted to be pale.

For each patient below, choose the most likely diagnosis from the list of options.
A.	Cerebrovascular accident
B.	Myasthenia gravis
C.	Carcinoma of oesophagus
D.	Hiatus hernia
E.	Achalasia
F.	Plummer-Vinson syndrome
G.	Pharyngeal pouch
H.	Gastric volvulus
I.	Carcinoma of bronchus
J.	Pneumonia
K.	Thyroid goitre
A

F. Plummer-Vinson syndrome

Plummer-Vinson syndrome is the association of chronic IDA (shown here by the koilonychia and paleness on examination) with dysphagia due to a post cricoid web. Roughly 7% of those with IDA may complain of gradual onset dysphagia with the discomfort found in the area of the cricoid cartilage. Invasive procedures may be needed for management such as endoscopic dilation of the web but treatment is largely aimed at correcting the IDA.

88
Q

A 40 year old man presents with dysphagia that worsens as he eats. He has droopy eyelids & sometimes has difficulty in speaking.

For each patient below, choose the most likely diagnosis from the list of options.
A.	Cerebrovascular accident
B.	Myasthenia gravis
C.	Carcinoma of oesophagus
D.	Hiatus hernia
E.	Achalasia
F.	Plummer-Vinson syndrome
G.	Pharyngeal pouch
H.	Gastric volvulus
I.	Carcinoma of bronchus
J.	Pneumonia
K.	Thyroid goitre
A

B. Myasthenia gravis

Myasthenia gravis is an autoimmune condition with antibodies affecting the NMJ, mostly the nAChR at the post-synaptic muscle membrane. Although some have antibodies against MuSK, and there are other proteins involved. MG is characterised by muscle weakness which increases with exercise (fatigue, demonstrated here as the dysphagia gets worse as he eats, unlike Lambert-Eaton myasthenic syndrome). Commonly, presentations include diplopia and drooping eyelids like this patient, and there may also be SOB, proximal limb weakness, facial paresis and oropharyngeal weakness. MG is associated with thymic hyperplasia in 70% or thymoma in 10%, and these associations can also crop up in EMQs. There will be elevated serum AChR receptor antibody titres or MuSK antibodies. Electrophysiology will demonstrate a decremental response on repetitive nerve stimulation. Treatment includes anticholinesterases (pyridostigmine, and immunotherapy. Patients may also require a thymectomy. Some 15-20% may experience a myasthenic crisis (which needs mechanical ventilation). Do you know what the Tensilon test is and why edrophonium is given in this test?

89
Q

A 50 year old describes a 5 month history of heartburn and cramp-like chest pain relived by drinking cold water, both unrelated to food. There has also been intermittent dysphagia to both liquids and solids, regurgitation and weight loss of 2kg.

For each patient below, choose the most likely diagnosis from the list of options.
A.	Cerebrovascular accident
B.	Myasthenia gravis
C.	Carcinoma of oesophagus
D.	Hiatus hernia
E.	Achalasia
F.	Plummer-Vinson syndrome
G.	Pharyngeal pouch
H.	Gastric volvulus
I.	Carcinoma of bronchus
J.	Pneumonia
K.	Thyroid goitre
A

E. Achalasia

This is achalasia which is a motility disorder with loss of peristalsis in the distal oesophagus and failure of the LOS to relax in response to swallowing. This presents commonly with dysphagia to both liquids and solids, regurgitation and retrosternal chest pain, which can be slowly progressive over time. In structural obstruction such as cancer, dysphagia to liquids is uncommon unless the disease is very advanced. Retrosternal pressure experienced can be precipitated by drinking liquids but is eased by continuing to drink, and the pain may be relieved by cold water. This may wake the individual from sleep. A UGI endoscopy is needed to exclude malignancy as a cause of dysphagia. The diagnosis is established on manometry or barium studies. Treatment is symptomatic.

90
Q

A 45 year old lady presenting with intense pruritus, joint pains & tiredness. She has also noticed that she is becoming yellow. On examination she was clubbed with xanthelasma around her eyes. She has dry eyes and a dry mouth. Her spleen was also palpably enlarged.

For each patient below, choose the SINGLE most likely cause of the symptom from the list of options.
A.	Duodenal ulcer
B.	Carcinoma of oesophagus
C.	Inflammatory bowel disease
D.	Irritable bowel syndrome
E.	Hiatus hernia
F.	Primary biliary cirrhosis
G.	Cancer of the liver
H.	Gastric ulcer
I.	Liver cirrhosis
J.	Carcinoma of oesophagus
K.	Pancreatitis
L.	Chronic hepatitis
M.	Coeliac’s disease
N.	Cancer of the pancreas
O.	Reflux oesophagitis
A

F. Primary biliary cirrhosis

Primary biliary cirrhosis (PBC) is a chronic condition where the intrahepatic small bile ducts are progressively damaged (and eventually lost) occuring on a background of portal tract inflammation. Fibrosis develops, ultimately leading to cirrhosis (which is defined as fibrosis with nodular regeneration). It is widely believed to be autoimmune in aetiology as almost all patients have AMA. The pointers in this question which would raise your suspicion, is xanthelasma around the eyes, pruritis in the absence of an obvious dermatological cause, fatigue and the features of liver disease typical of cirrhosis (jaundice) and splenomegaly as a feature of portal hypertension. The patient also has dry eyes and dry mouth from associated Sjogren’s syndrome. The joint pains could indicate RA.

91
Q

A 70 year old man presents with general malaise, weakness & right upper quadrant abdominal pain. On examination he looked ill & was clinically jaundiced. There were spider naevi, palmar erythema, leuconychia, Dupuytren’s contracture & gynaecomastia.

For each patient below, choose the SINGLE most likely cause of the symptom from the list of options.
A.	Duodenal ulcer
B.	Carcinoma of oesophagus
C.	Inflammatory bowel disease
D.	Irritable bowel syndrome
E.	Hiatus hernia
F.	Primary biliary cirrhosis
G.	Cancer of the liver
H.	Gastric ulcer
I.	Liver cirrhosis
J.	Carcinoma of oesophagus
K.	Pancreatitis
L.	Chronic hepatitis
M.	Coeliac’s disease
N.	Cancer of the pancreas
O.	Reflux oesophagitis
A

I. Liver cirrhosis

Cirrhosis is the end-stage of chronic liver disease, resulting in hepatic insufficiency and portal hypertension. This has resulted in this man’s jaundice. Palmar erythema affects the thenar and hypothenar eminences. Other signs include spider naevi, telangiectasia, Dupuytren’s contracture, parotid swelling, leuconychia from hypoalbuminaemia, gynaecomastia and bruising. Management is aimed at treating the underlying liver disease. The only curative option, once decompensated, is liver transplantation.

92
Q

A 25 year old lady presented with fever, abdominal pains & weight loss. She was opening her bowels x 10-12/day with blood & mucus.

For each patient below, choose the SINGLE most likely cause of the symptom from the list of options.
A.	Duodenal ulcer
B.	Carcinoma of oesophagus
C.	Inflammatory bowel disease
D.	Irritable bowel syndrome
E.	Hiatus hernia
F.	Primary biliary cirrhosis
G.	Cancer of the liver
H.	Gastric ulcer
I.	Liver cirrhosis
J.	Carcinoma of oesophagus
K.	Pancreatitis
L.	Chronic hepatitis
M.	Coeliac’s disease
N.	Cancer of the pancreas
O.	Reflux oesophagitis
A

C. Inflammatory bowel disease

This patient gives a history of IBD. The history would be more suggestive of UC (but could be CD) where the mainstay of treatment is with 5-ASA. A colonoscopy is required to assess the extent of disease and for a definitive diagnosis. Biopsy in CD will show transmural granulomatous inflammation. CD can affect the whole GIT but favours the TI and proximal colon and is macroscopically characterised by skip lesions. UC on the other hand is characterised by the presence of crypt abscesses, which is pathognomic. CD risk is increased 3-4 fold by smoking whereas smoking seems protective in UC. The mainstay of treatment in CD is with steroids and azathioprine to revent relapses and for those suffering side effects of steroid treatment. TNF-alpha inhibitors also have a role. Surgery in CD is only indicated in a small number of patients who bleed, for bowel perforation and cases of complete obstruction. The aim is to rest distal disease by temporarily diverting faecal flow.

93
Q

A 40 year old anxious lady with intermittent abdominal pain relieved by defecation. Her abdomen feels distended & her stool, alternated between diarrhoea & constipation.

For each patient below, choose the SINGLE most likely cause of the symptom from the list of options.
A.	Duodenal ulcer
B.	Carcinoma of oesophagus
C.	Inflammatory bowel disease
D.	Irritable bowel syndrome
E.	Hiatus hernia
F.	Primary biliary cirrhosis
G.	Cancer of the liver
H.	Gastric ulcer
I.	Liver cirrhosis
J.	Carcinoma of oesophagus
K.	Pancreatitis
L.	Chronic hepatitis
M.	Coeliac’s disease
N.	Cancer of the pancreas
O.	Reflux oesophagitis
A

D. Irritable bowel syndrome

IBS is a chronic condition with abdominal pain associated with bowel dysfunction and is a diagnosis of exclusion. The pain or discomfort may be relieved by defecation. Examination is usually unremarkable and the diagnosis is based on the patient’s history in line with the Rome Criteria. If the patient presents with any worrying symptoms, then these will warrant a more thorough investigation. Treatment depends on the patient’s predominant symptoms. Antispasmodics relieve abdominal pain or discomfort but do not affect bowel habit. Examples include peppermint oil and dicycloverine. Laxatives can also be used such as lactulose. Lifestyle and dietary modifications combined with reassurance remain the 1st line intervention for functional bowel disease. IBS is linked with stressful jobs such as working as a secretary and there is a female/male ratio of 2:1. ‘Pellet-like’ stools is an indicator in EMQs.

94
Q

A 70 year old diabetic man, who smoked 40 cigs/day for 40 years. He presents with abdominal pain worse at night & radiating to his back. He is losing weight, suffers from dyspepsia & pruritus. On examination he is cachectic, jaundiced & has an enlarged gallbladder.

For each patient below, choose the SINGLE most likely cause of the symptom from the list of options.
A.	Duodenal ulcer
B.	Carcinoma of oesophagus
C.	Inflammatory bowel disease
D.	Irritable bowel syndrome
E.	Hiatus hernia
F.	Primary biliary cirrhosis
G.	Cancer of the liver
H.	Gastric ulcer
I.	Liver cirrhosis
J.	Carcinoma of oesophagus
K.	Pancreatitis
L.	Chronic hepatitis
M.	Coeliac’s disease
N.	Cancer of the pancreas
O.	Reflux oesophagitis
A

N. Cancer of the pancreas

Pancreatic cancer typically presents with painless obstructive jaundice and weight loss and generally presents late. It can however, like this case, present with abdominal pain which is typically non-specific in the upper abdomen. If the patient presents with persistent back pain, then this symptom is consistent with retroperitoneal metastases. It is estimated that 1 in 4 cases can be linked to smoking. Whipple’s procedure or Traverso-Longmire procedure (pancreaticoduodenectomy) offers the only hope of a cure but only a small minority are elegible for these procedures. The first tests to order are an abdominal USS and LFTs. Note Courvoisier’s law with regard to this question: Jaundice and a palpable painless gallbladder is unlikely to be caused by gallstones. The tumour marker for pancreatic cancer is CA19-9 which is useful in preoperative staging.

95
Q

A 50 year old man: BR 50umol/l, ALP 200iu/l, ALT 120iu/l, GGT 600iu/l.

For each set of test results below choose the most likely cause of jaundice from the list of options.
A.	Acute pancreatitis
B.	Cholangiocarcinoma
C.	Gilbert’s syndrome
D.	Gallstones
E.	Hepatocellular carcinoma
F.	Wilson’s disease
G.	Paracetamol overdose
H.	Drug-induced cholestasis
I.	Viral hepatitis
J.	CMV infection
K.	Pancreatic cancer
L.	Alcoholic cirrhosis
M.	Haemolysis
A

L. Alcoholic cirrhosis

Cirrhosis is the end-stage of chronic liver disease, in this case due to alcoholic liver disease. Cirrhosis results in hepatic insufficiency and portal hypertension. The high GGT here is as a result of high alcohol consumption.

Liver function tests are mistakenly named really because they don’t test liver function. They are better called ‘liver tests’ or ‘tests of liver chemistry’. The tests of liver synthetic function come from measuring albumin levels (produced by the liver) and the prothrombin time, as the liver has a key role in the manufacture of clotting factors. Some general points on liver tests to consider: These tests tend to include bilirubin (breakdown product of RBC after hepatic conjugation, and is secreted in the biliary system), AST, ALT, GGT, ALP (alk phos) and LDH. It’s really all about pattern recognition. Isolated elevation of liver tests tends to make you think of non-hepatic causes and you should take into account the patient’s history during interpretation. A normal liver panel does not mean that the person does not have liver disease, and liver tests are elevated in a small percentage of normals.

You can split the possible causes into three broad categories. It is worth noting that people with any pattern can have cirrhosis. AST is also present in heart, skeletal muscle, kidney, brain and in RBCs too. 80% of AST is found in the mitochondria and 20% in the cytoplasmic matrix. ALP, which is a cytoplasmic enzyme, can come from bone, kidney, intestines or the placenta. GGT may come from the heart or RBCs.

The first is predominantly elevated AST/ALT, which is described as a hepatocellular pattern and occurs in conditions such as viral hepatitis, alcoholic disease, Wilson’s etc. Here the hepatocytes get damaged and enzymes leak out.

The second is predominantly elevated ALP (GGT too, but an isolated rise in GGT is common so many hospitals remove GGT from the liver panel. GGT is only useful if there is an isolated rise in ALP). This is seen in a cholestatic (here, the hepatobiliary system is affected) or infiltrative pattern in bile duct obstruction, malignancy e.g. HCC or pancreatic, PBC/PSC etc or infiltrative conditions such as TB and lymphomas.

The last is an isolated hyperbilirubinaemia seen in haemolysis, intra-abdominal bleeding or with conditions such as Gilbert’s. BR elevation can be further divided into conjugated or unconjugated. Unconjugated is normally due to breakdown of RBC beyond the liver’s capacity to conjugate. Conjugated occurs in liver disease and problems with obstructed bile flow.

96
Q

A 50 year old man: BR 110umol/l, ALP 300iu/l, ALT 110iu/l, AFP is elevated.

For each set of test results below choose the most likely cause of jaundice from the list of options.
A.	Acute pancreatitis
B.	Cholangiocarcinoma
C.	Gilbert’s syndrome
D.	Gallstones
E.	Hepatocellular carcinoma
F.	Wilson’s disease
G.	Paracetamol overdose
H.	Drug-induced cholestasis
I.	Viral hepatitis
J.	CMV infection
K.	Pancreatic cancer
L.	Alcoholic cirrhosis
M.	Haemolysis
A

E. Hepatocellular carcinoma

The pattern here is cholestatic. Furthermore, elevated AFP is given, which is a tumour marker for HCC, differentiating this from other potential causes like pancreatic cancer. AST and ALT may be normal or elevated.

Liver function tests are mistakenly named really because they don’t test liver function. They are better called ‘liver tests’ or ‘tests of liver chemistry’. The tests of liver synthetic function come from measuring albumin levels (produced by the liver) and the prothrombin time, as the liver has a key role in the manufacture of clotting factors. Some general points on liver tests to consider: These tests tend to include bilirubin (breakdown product of RBC after hepatic conjugation, and is secreted in the biliary system), AST, ALT, GGT, ALP (alk phos) and LDH. It’s really all about pattern recognition. Isolated elevation of liver tests tends to make you think of non-hepatic causes and you should take into account the patient’s history during interpretation. A normal liver panel does not mean that the person does not have liver disease, and liver tests are elevated in a small percentage of normals.

You can split the possible causes into three broad categories. It is worth noting that people with any pattern can have cirrhosis. AST is also present in heart, skeletal muscle, kidney, brain and in RBCs too. 80% of AST is found in the mitochondria and 20% in the cytoplasmic matrix. ALP, which is a cytoplasmic enzyme, can come from bone, kidney, intestines or the placenta. GGT may come from the heart or RBCs.

The first is predominantly elevated AST/ALT, which is described as a hepatocellular pattern and occurs in conditions such as viral hepatitis, alcoholic disease, Wilson’s etc. Here the hepatocytes get damaged and enzymes leak out.

The second is predominantly elevated ALP (GGT too, but an isolated rise in GGT is common so many hospitals remove GGT from the liver panel. GGT is only useful if there is an isolated rise in ALP). This is seen in a cholestatic (here, the hepatobiliary system is affected) or infiltrative pattern in bile duct obstruction, malignancy e.g. HCC or pancreatic, PBC/PSC etc or infiltrative conditions such as TB and lymphomas.

The last is an isolated hyperbilirubinaemia seen in haemolysis, intra-abdominal bleeding or with conditions such as Gilbert’s. BR elevation can be further divided into conjugated or unconjugated. Unconjugated is normally due to breakdown of RBC beyond the liver’s capacity to conjugate. Conjugated occurs in liver disease and problems with obstructed bile flow.

97
Q

A 35 year old woman: BR 80umol/l, ALP 300iu/l, ALT 30iu/l, GGT 30iu/l. Abdominal USS shows biliary dilation. The patient has RUQ tenderness.

For each set of test results below choose the most likely cause of jaundice from the list of options.
A.	Acute pancreatitis
B.	Cholangiocarcinoma
C.	Gilbert’s syndrome
D.	Gallstones
E.	Hepatocellular carcinoma
F.	Wilson’s disease
G.	Paracetamol overdose
H.	Drug-induced cholestasis
I.	Viral hepatitis
J.	CMV infection
K.	Pancreatic cancer
L.	Alcoholic cirrhosis
M.	Haemolysis
A

D. Gallstones

This is again a cholestatic pattern and the ultrasound scan means this is either choledocholithiasis, pancreatic cancer or cholangiocarcinoma. The fact there is RUQ pain points more towards gallstones. The other two tend to present painlessly.

Liver function tests are mistakenly named really because they don’t test liver function. They are better called ‘liver tests’ or ‘tests of liver chemistry’. The tests of liver synthetic function come from measuring albumin levels (produced by the liver) and the prothrombin time, as the liver has a key role in the manufacture of clotting factors. Some general points on liver tests to consider: These tests tend to include bilirubin (breakdown product of RBC after hepatic conjugation, and is secreted in the biliary system), AST, ALT, GGT, ALP (alk phos) and LDH. It’s really all about pattern recognition. Isolated elevation of liver tests tends to make you think of non-hepatic causes and you should take into account the patient’s history during interpretation. A normal liver panel does not mean that the person does not have liver disease, and liver tests are elevated in a small percentage of normals.

You can split the possible causes into three broad categories. It is worth noting that people with any pattern can have cirrhosis. AST is also present in heart, skeletal muscle, kidney, brain and in RBCs too. 80% of AST is found in the mitochondria and 20% in the cytoplasmic matrix. ALP, which is a cytoplasmic enzyme, can come from bone, kidney, intestines or the placenta. GGT may come from the heart or RBCs.

The first is predominantly elevated AST/ALT, which is described as a hepatocellular pattern and occurs in conditions such as viral hepatitis, alcoholic disease, Wilson’s etc. Here the hepatocytes get damaged and enzymes leak out.

The second is predominantly elevated ALP (GGT too, but an isolated rise in GGT is common so many hospitals remove GGT from the liver panel. GGT is only useful if there is an isolated rise in ALP). This is seen in a cholestatic (here, the hepatobiliary system is affected) or infiltrative pattern in bile duct obstruction, malignancy e.g. HCC or pancreatic, PBC/PSC etc or infiltrative conditions such as TB and lymphomas.

The last is an isolated hyperbilirubinaemia seen in haemolysis, intra-abdominal bleeding or with conditions such as Gilbert’s. BR elevation can be further divided into conjugated or unconjugated. Unconjugated is normally due to breakdown of RBC beyond the liver’s capacity to conjugate. Conjugated occurs in liver disease and problems with obstructed bile flow.

98
Q

A 20 year old man: BR 45umol/l, (Conjugated 7, unconjugated 38), ALP 40iu/l, AST 12iu/l, Hb 15g/dl, Normal blood film

For each set of test results below choose the most likely cause of jaundice from the list of options.
A.	Acute pancreatitis
B.	Cholangiocarcinoma
C.	Gilbert’s syndrome
D.	Gallstones
E.	Hepatocellular carcinoma
F.	Wilson’s disease
G.	Paracetamol overdose
H.	Drug-induced cholestasis
I.	Viral hepatitis
J.	CMV infection
K.	Pancreatic cancer
L.	Alcoholic cirrhosis
M.	Haemolysis
A

C. Gilbert’s syndrome

Gilbert’s occurs in an asymptomatic patient, often as an incidental finding or mild jaundice occuring in adolescence/young adult age. There is elevated unconjugated BR with other liver tests being normal. The blood smear is also normal with normal reticulocyte count, and normal Hb indicating that this is not due to haemolysis. It is a common syndrome and is not really a disease, more a physiological variant. No treatment is needed and this condition is due to decreased UDPGT activity leading to decreased conjugation of unconjugated bilirubin, leading to elevated levels.

Liver function tests are mistakenly named really because they don’t test liver function. They are better called ‘liver tests’ or ‘tests of liver chemistry’. The tests of liver synthetic function come from measuring albumin levels (produced by the liver) and the prothrombin time, as the liver has a key role in the manufacture of clotting factors. Some general points on liver tests to consider: These tests tend to include bilirubin (breakdown product of RBC after hepatic conjugation, and is secreted in the biliary system), AST, ALT, GGT, ALP (alk phos) and LDH. It’s really all about pattern recognition. Isolated elevation of liver tests tends to make you think of non-hepatic causes and you should take into account the patient’s history during interpretation. A normal liver panel does not mean that the person does not have liver disease, and liver tests are elevated in a small percentage of normals.

You can split the possible causes into three broad categories. It is worth noting that people with any pattern can have cirrhosis. AST is also present in heart, skeletal muscle, kidney, brain and in RBCs too. 80% of AST is found in the mitochondria and 20% in the cytoplasmic matrix. ALP, which is a cytoplasmic enzyme, can come from bone, kidney, intestines or the placenta. GGT may come from the heart or RBCs.

The first is predominantly elevated AST/ALT, which is described as a hepatocellular pattern and occurs in conditions such as viral hepatitis, alcoholic disease, Wilson’s etc. Here the hepatocytes get damaged and enzymes leak out.

The second is predominantly elevated ALP (GGT too, but an isolated rise in GGT is common so many hospitals remove GGT from the liver panel. GGT is only useful if there is an isolated rise in ALP). This is seen in a cholestatic (here, the hepatobiliary system is affected) or infiltrative pattern in bile duct obstruction, malignancy e.g. HCC or pancreatic, PBC/PSC etc or infiltrative conditions such as TB and lymphomas.

The last is an isolated hyperbilirubinaemia seen in haemolysis, intra-abdominal bleeding or with conditions such as Gilbert’s. BR elevation can be further divided into conjugated or unconjugated. Unconjugated is normally due to breakdown of RBC beyond the liver’s capacity to conjugate. Conjugated occurs in liver disease and problems with obstructed bile flow.

99
Q

A 35 year old lawyer with a history of recurring piles & having been treated by his GP with sclerosing therapy continues to bleed. He is referred to surgical outpatients for more definitive treatment.

For each of the patients below, choose the most likely management from the list of options.
A.	Topical GTN
B.	Anterior resection
C.	Colostomy
D.	IV corticosteroids
E.	Loperamide (Imodium)
F.	Haemorrhoidectomy
G.	High fibre diet
H.	Blood transfusion
I.	Palliative care
J.	IV immunoglobulin
K.	Hemicolectomy
A

F. Haemorrhoidectomy

Haemorrhoidectomy is the treatment of choice of choice for patients with grade 4 haemorrhoids or for any patient who has failed with more conservative treatment such as sclerotherapy. Grade 1 is limited to within the anal canal. Grade 2 protrudes but spontaneously reduces when the patient stops straining. Grade 3 protrudes and reduces fully on manual pressure. Grade 4 is irreducible. Lord’s anal stretch is no longer recommended due to the risk of faecal incontinence.

100
Q

A 33 year old man has severe pain and fresh bleeding on defecation.

For each of the patients below, choose the most likely management from the list of options.
A.	Topical GTN
B.	Anterior resection
C.	Colostomy
D.	IV corticosteroids
E.	Loperamide (Imodium)
F.	Haemorrhoidectomy
G.	High fibre diet
H.	Blood transfusion
I.	Palliative care
J.	IV immunoglobulin
K.	Hemicolectomy
A

A. Topical GTN

This patient has an anal fissure. This causes severe pain on defecation and may continue for 1 to 2 hours. A small amount of fresh blood is often passed on the stool. Initial treatment is with topical GTN or diltiazem along with supportive measures such a high fibre diet. Resistant or chronic fissures may benefit from surgical measures or botulinum toxin.

101
Q

A 39 year old woman who is on mesalazine for her ulcerative colitis presents with 12 daily bowel movements and massive continuous bleeding PR; she is noted to have a raised ESR.

For each of the patients below, choose the most likely management from the list of options.
A.	Topical GTN
B.	Anterior resection
C.	Colostomy
D.	IV corticosteroids
E.	Loperamide (Imodium)
F.	Haemorrhoidectomy
G.	High fibre diet
H.	Blood transfusion
I.	Palliative care
J.	IV immunoglobulin
K.	Hemicolectomy
A

D. IV corticosteroids

IV corticosteroids are used in fulminant disease. These patients need to be admitted. If there is no response to IV corticosteroids within 24 to 48 hours then surgery is indicated.

102
Q

A 24 year old woman following a viral infection was diagnosed as having idiopathic thrombocytopaenia. She presents to A&E & complains of multiple bruising & rectal bleeding. She is on oral prednisolone 30mg/day. Her Hb is 12.5g/dl.

For each of the patients below, choose the most likely management from the list of options.
A.	Topical GTN
B.	Anterior resection
C.	Colostomy
D.	IV corticosteroids
E.	Loperamide (Imodium)
F.	Haemorrhoidectomy
G.	High fibre diet
H.	Blood transfusion
I.	Palliative care
J.	IV immunoglobulin
K.	Hemicolectomy
A

J. IV immunoglobulin

ITP is thought to be due to an autoimmune phenomenon. Treatment is based on the patient’s platelet count and bleeding symptoms. This patient has severe active bleeding and must be started on IVIG plus corticosteroids, which she is already on. Platelet transfusions should be considered with tranexamic acid as an adjunct.

103
Q

A 58 year old man was admitted complaining of abdominal pain. He is found to have rectal carcinoma.

For each of the patients below, choose the most likely management from the list of options.
A.	Topical GTN
B.	Anterior resection
C.	Colostomy
D.	IV corticosteroids
E.	Loperamide (Imodium)
F.	Haemorrhoidectomy
G.	High fibre diet
H.	Blood transfusion
I.	Palliative care
J.	IV immunoglobulin
K.	Hemicolectomy
A

B. Anterior resection

Treatment of rectal carcinoma involves surgical excision where possible. This can either be an anterior resection (tumours in the upper 1/3 of the rectum) or an abdominoperineal resection (if the tumour lies lower down). APER involves the formation of a permanent colostomy and has a high incidence of sexual and urinary dysfunction. Anterior resection involves a colo-anal anastamosis.

104
Q

A 64 year old man presents with a 1 day history of dark rectal bleeding. He has complained of some diarrhoea for the last 6 months, & has noticed some loss of weight. Rectal examination & proctoscopy are normal.

The patients have all presented with rectal bleeding. Please select the most appropriate diagnosis from the list.
A.	Ischaemic colitis
B.	Haemorrhoids
C.	Diverticular disease
D.	Crohn’s disease
E.	Anal carcinoma
F.	Ulcerative colitis
G.	Colonic polyp
H.	Anal fissure
I.	Colonic carcinoma
J.	Infective colitis
A

I. Colonic carcinoma

The symptoms of colorectal cancer are non-specific but the GI symptoms here combined with weight loss should make you suspicious. Symptoms include change in bowel habit, rectal bleeding and anaemia, commonly associated with right sided cancer. Distension, weight loss and vomiting are usually associated with advanced disease. Colonoscopy will be needed in this patient for diagnosis and a biopsy of any suspicious lesion found. Main curative treatment is surgical resection. CEA is the classic tumour marker, which is raised in about 80% but is not really sensitive or specific enough to be useful in diagnosis or screening, but is used to monitor treatment and recurrence in those who have already been diagnosed.

105
Q

A 24 year old woman presents with a 3 month history of lower abdominal colicky pain, diarrhoea (bowels open 6-10 times per day) & passage of blood mixed with the stool.

The patients have all presented with rectal bleeding. Please select the most appropriate diagnosis from the list.
A.	Ischaemic colitis
B.	Haemorrhoids
C.	Diverticular disease
D.	Crohn’s disease
E.	Anal carcinoma
F.	Ulcerative colitis
G.	Colonic polyp
H.	Anal fissure
I.	Colonic carcinoma
J.	Infective colitis
A

F. Ulcerative colitis

While this could be Crohn’s disease, bloody diarrhoea is more commonly a presentation of UC than Crohn’s. UC is characterised by diffuse mucosal inflammation running a relapsing and remitting course. Bloody diarrhoea is commonly experienced by patients who may also complain of other symptoms such as (lower) abdominal pain, faecal urgency and the host of extra-intestinal manifestations associated with UC. Diagnosis of UC requires endoscopy with biopsy and a negative stool culture to rule out infectious gastroenteritis. Flare ups are usually linked to pathogens so a stool culture will always be needed in these cases. Toxic megacolon is a complication which is associated with a risk of perforation. UC is also linked with bowel adenocarinoma and PSC. Treatment involves mesalazine (5-ASA) used to induce and maintain remission.

106
Q

A 54 year old man with no previous abdominal symptoms complains of several episodes of painless bright red rectal bleeding which is separate from the stool. Abdominal, rectal examination & proctoscopy are normal.

The patients have all presented with rectal bleeding. Please select the most appropriate diagnosis from the list.
A.	Ischaemic colitis
B.	Haemorrhoids
C.	Diverticular disease
D.	Crohn’s disease
E.	Anal carcinoma
F.	Ulcerative colitis
G.	Colonic polyp
H.	Anal fissure
I.	Colonic carcinoma
J.	Infective colitis
A

G. Colonic polyp

Most polyps are asymptomatic and rectal bleeding is indeed an unusual symptom to present with (though FOBT may be positive), but the lack of previous symptoms and normal examination findings makes this likely to be a bleeding polyp. The next step here would be endoscopy with polypectomy for histological examination. Indeed they are frequently found incidentally on screening with FOBT or colonoscopy for another reason. If there are a few polyps, all of them will be removed. If there are many, a sample will be removed for biopsy. Polyps can also be inherited in FAP, also seen in Gardner syndrome and Peutz-Jeghers. They can be neoplastic such as adenomas or non-neoplastic such as hyperplastic polyps.

107
Q

A 24 year old man presents with a 3 month history of episodes of painless, bright red rectal bleeding on straining at stool. He has noticed some blood in the bowl, separate from the stool & some on the paper after wiping.

The patients have all presented with rectal bleeding. Please select the most appropriate diagnosis from the list.
A.	Ischaemic colitis
B.	Haemorrhoids
C.	Diverticular disease
D.	Crohn’s disease
E.	Anal carcinoma
F.	Ulcerative colitis
G.	Colonic polyp
H.	Anal fissure
I.	Colonic carcinoma
J.	Infective colitis
A

B. Haemorrhoids

Haemorrhoids are vascular rich cushions in the anal canal and presents, typically, as painless bright PR bleeding or with sudden onset pain in the area associated with a palpable mass. Pruritus ani is common and there is often perianal pain or discomfort. Diagnosis is made visually. Grade 1 is limited to within the anal canal. Grade 2 protrudes but spontaneously reduces when the patient stops straining. Grade 3 protrudes and reduces fully on manual pressure. Grade 4 is irreducible. Treatment includes fibre, ligation, photocoagulation, sclerotherapy or surgical haemorrhoidectomy. Haemorrhoidectomy is the treatment of choice of choice for patients with grade 4 haemorrhoids or for any patient who has failed with more conservative treatment such as sclerotherapy.

108
Q

A 28 year old female medical student returns from her elective in Africa with a short history of severe lower abdominal cramps & the passage of bloody diarrhoea.

The patients have all presented with rectal bleeding. Please select the most appropriate diagnosis from the list.
A.	Ischaemic colitis
B.	Haemorrhoids
C.	Diverticular disease
D.	Crohn’s disease
E.	Anal carcinoma
F.	Ulcerative colitis
G.	Colonic polyp
H.	Anal fissure
I.	Colonic carcinoma
J.	Infective colitis
A

J. Infective colitis

This patient has recent foreign travel and a short history, previously normal, making infectious diarrhoea very likely. Probably eaten something dodgy while on elective. Think of the organisms which cause bloody diarrhoea such as EHEC. E coli is the most common cause of traveller’s diarrhoea. There may be a contact history due to faecal-oral spread. The mainstay of treatment is rehydration and supportive therapy. Antibiotics may be indicated, particularly in severe ETEC.

109
Q

A 55 year old man presents with a few months history of weight loss, lethargy & fever. On examination, he has a large liver & spleen. His WBC is 10.2x109/l & his blood film shows increased granulocytes & granulocyte precursors.

For each patient with splenomegaly, select the most likely diagnosis from the options.
A. Infectious mononucleosis
B. Sarcoidosis
C. Gaucher’s disease
D. Portal hypertension
E. Malaria
F. CML
G. IDA
H. Hodgkin’s disease
I. Idiopathic myelofibrosis
J. Polycythaemia
K. Metastatic carcinoma
L. Bacterial endocarditis
A

F. CML

This is CML which tends to present in th 30-60 age group. At presentation 1/3 may be asymptomatic though if symptomatic, it presents with symptoms including fever, weight loss and night sweats. There is myeloid stem cell proliferation and presents with raised neutrophils, metamyelocytes and basophils. The patient’s granulocytosis is suggestive of CML. CML is associated with the philadelphia chromosome characterised by t(9;22) of bcr-abl. There tends to be massive splenomegaly which is the most common physical finding on examination. This conditon may transform to AML or ALL in what is known as a ‘blast crisis’. CML responds to imatinib, which is an anti-bcr-abl antibody and gives long term remission in most patients.

110
Q

A 27 year old Afro-Caribbean man presents with fever, weight loss and an intractable itch. His spleen is just palpable and he has two 3cm nodes in his right neck. Hb is low.

For each patient with splenomegaly, select the most likely diagnosis from the options.
A. Infectious mononucleosis
B. Sarcoidosis
C. Gaucher’s disease
D. Portal hypertension
E. Malaria
F. CML
G. IDA
H. Hodgkin’s disease
I. Idiopathic myelofibrosis
J. Polycythaemia
K. Metastatic carcinoma
L. Bacterial endocarditis
A

H. Hodgkin’s disease

This is a case of lymphoma. Reed-Sternberg cells are binucleate cells characteristically seen in Hodgkin’s lymphoma. Hodgkin’s is localised to a single group of nodes (normally the cervical and/or supraclavicular) and extranodal involvement is rare. Mediastinal involvement is common. Spread is contiguous and B symptoms may be present such as a low grade fever, weight loss and night sweats. Pruritis may be found in approximately 10% of cases but has no prognostic significance. 50% of cases is associated with EBV infection and distribution is bimodal with peaks in young and old. There is classically pain in lymph nodes on alcohol consumption.

111
Q

An 18 year old Caucasian shop assistant presents with fever & a sore throat. She is found to have enlarged but soft cervical lymph nodes & a soft spleen palpable 3cm below the costal margin. Blood film shows atypical lymphocytes.

For each patient with splenomegaly, select the most likely diagnosis from the options.
A. Infectious mononucleosis
B. Sarcoidosis
C. Gaucher’s disease
D. Portal hypertension
E. Malaria
F. CML
G. IDA
H. Hodgkin’s disease
I. Idiopathic myelofibrosis
J. Polycythaemia
K. Metastatic carcinoma
L. Bacterial endocarditis
A

A. Infectious mononucleosis

This is caused by EBV and characterised by fever, pharyngitis and lymphadenopathy with atypical lymphocytosis. Positive heterophile antibody test and serological testing for EBV antibodies are diagnostic. Splenomegaly is common and enlargement occurs in the first week, lasting 3-4 weeks. It is worth remembering that splenomegaly is always an abnormal examination finding. IM is commonly named the ‘kissing’ disease as EBV is most commonly transmitted by saliva. Penetrative sex and general promiscuity in young women also increases the risk.

112
Q

A 70 year old English woman presents with fever, slight splenomegaly, splinter haemorrhages & a diastolic murmur at the apex of the heart.

For each patient with splenomegaly, select the most likely diagnosis from the options.
A. Infectious mononucleosis
B. Sarcoidosis
C. Gaucher’s disease
D. Portal hypertension
E. Malaria
F. CML
G. IDA
H. Hodgkin’s disease
I. Idiopathic myelofibrosis
J. Polycythaemia
K. Metastatic carcinoma
L. Bacterial endocarditis
A

L. Bacterial endocarditis

Any patient presenting with fever and a new murmur should always make you think of bacterial endocarditis. The classic new or worsening murmur is actually rare. As are splinter haemorrhages, which this patient has. Other uncommon signs you may find include Janeway lesions (painless macular haemorrhagic plaques on the palms and soles) and Osler nodes (painful nodules on the pads of the fingers and toes). Roth spots may also be seen on fundoscopy. Three sets of bood cultures are required and this patient will have to go for an echocardiogram.The Duke criteria is used for diagnosis.

113
Q

A 65 year old man presents with angina & claudication. He is found to have a firm spleen extending 20cm below the costal margin. His Hb is 7.5g/dl & his blood film is leuco-erythroblastic.

For each patient with splenomegaly, select the most likely diagnosis from the options.
A. Infectious mononucleosis
B. Sarcoidosis
C. Gaucher’s disease
D. Portal hypertension
E. Malaria
F. CML
G. IDA
H. Hodgkin’s disease
I. Idiopathic myelofibrosis
J. Polycythaemia
K. Metastatic carcinoma
L. Bacterial endocarditis
A

I. Idiopathic myelofibrosis

This is a case of myelofibrosis. Leucoerythroblastosis and splenomegaly are common findings. Strong risk factors include exposure to radiation and industrial solvents. BM biopsy is essential for diagnosis. Extramedullary haematopoiesis leads to dacrocytes in the peripheral blood smear. Those without symptoms can be managed with folate and pyridoxine supplements. Otherwise options such as a BM transplant and hydroxycarbamide can be considered.